Chapter 1-6 Exam Questions_Robins & Cotran

¡Supera tus tareas y exámenes ahora con Quizwiz!

1 While shaving one morning, a 23-year-old man nicks his lip with a razor. Seconds after the injury, the bleeding stops. Which of the following mechanisms is most likely to reduce blood loss from a small dermal arteriole? □ (A) Protein C activation □ (B) Vasoconstriction □ (C) Platelet aggregation □ (D) Neutrophil chemotaxis □ (E) Fibrin polymerization

1 (B) The initial response to injury is arteriolar vasoconstriction, but this is transient, and the coagulation mechanism must be initiated to maintain hemostasis. Protein C is involved in anticoagulation to counteract clotting. Platelet aggregation occurs with release of factors such as ADP, but this takes several minutes. Neutrophils are not essential to hemostasis. Fibrin polymerization is part of secondary hemostasis after the vascular injury is initially closed.

A 17-year-old boy infected with hepatitis A experiences mild nausea for about 1 week and develops very mild scleral icterus. On physical examination, he has minimal right upper quadrant tenderness. Laboratory findings include a serum AST of 68 U/L, ALT of 75 U/L, and total bilirubin of 5.1 mg/dL. The increase in this patient's serum enzyme levels most likely results from which of the following changes in the hepatocytes? □ (A) Autophagy by lysosomes □ (B) Clumping of nuclear chromatin □ (C) Defects in the cell membrane □ (D) Dispersion of ribosomes □ (E) Swelling of the mitochondria

1 (C) Irreversible cell injury is associated with loss of membrane integrity. This allows intracellular enzymes to leak into the serum. All other morphologic changes listed are associated with reversible cell injury, in which the cell membrane remains intact.

1 In an experiment, surgical incisions are made in a study group of laboratory rats. Observations about the wounds are recorded over a 2-week period using various chemical mediators. Which of the following steps in the inflammatory-repair response is most likely affected by neutralization of transforming growth factor β (TGF-β)? □ (A) Leukocyte extravasation □ (B) Increase in vascular permeability □ (C) Production of collagen □ (D) Chemotaxis of lymphocytes □ (E) Migration of epithelial cells

1 (C) TGF-β stimulates many steps in fibrogenesis, including fibroblast chemotaxis and production of collagen by fibroblasts, while inhibiting degradation of collagen. All of the other steps listed are unaffected by TGF-β.

1 In a 6-month randomized trial of a pharmacologic agent, one group of patients receives a cyclooxygenase-2 (COX-2) inhibitor, and a control group does not. Laboratory measurements during the trial show no significant differences between the groups in WBC count, platelet count, hemoglobin, and creatinine. The group receiving the drug reports subjective findings different from those of the control group, however. Which of the following findings was most likely reported by the group receiving the drug? □ (A) Ankle swelling □ (B) Increased bouts of asthma □ (C) Easy bruisability □ (D) Reduced urticaria □ (E) Increased febrile episodes □ (F) Reduced arthritis pain

1 (F) The COX-2 enzyme is inducible with acute inflammatory reactions, particularly in neutrophils, in synovium, and in the central nervous system. The cyclooxygenase pathway of arachidonic acid metabolism generates prostaglandins, which mediate pain, fever, and vasodilation. Ankle swelling is most likely to result from peripheral edema secondary to congestive heart failure. Asthma results from bronchoconstriction mediated by leukotrienes that are generated by the lipoxygenase pathway of arachidonic acid metabolism. Easy bruisability results from prolonged glucocorticoid administration, which also causes leukopenia. Inhibition of histamine released from mast cells helps reduce urticaria. Fever can be mediated by prostaglandin release, not inhibition.

10 A 30-year-old man sustains a left femoral fracture in a skiing accident, and his leg is placed in a plaster cast. After the leg has been immobilized for several weeks, the diameter of the left calf has decreased. This change is most likely to result from which of the following alterations in the calf muscles? □ (A) Aplasia □ (B) Hypoplasia □ (C) Atrophy □ (D) Dystrophy □ (E) Hyalinosis

1 0 (C) Reduced workload causes shrinkage of cell size because of loss of cell substance, a process called atrophy. Aplasia refers to lack of embryonic development; hypoplasia describes poor or subnormal development. Dystrophy of muscles refers to inherited disorders of skeletal muscles that lead to muscle weakness and wasting. Hyaline change (hyalinosis) refers to a nonspecific, pink, glassy eosinophilic appearance of cells.

2 An experiment introduces bacteria into a perfused tissue preparation. Leukocytes leave the vasculature and migrate to the site of bacterial inoculation. The movement of these leukocytes is most likely to be mediated by which of the following substances? □ (A) Bradykinin □ (B) Chemokines □ (C) Histamine □ (D) Prostaglandins □ (E) Complement C3a

2 (B) Chemokines include many molecules that are chemotactic for neutrophils, eosinophils, lymphocytes, monocytes, and basophils. Bradykinin causes pain and increased vascular permeability. Histamine causes vascular leakage, and prostaglandins have multiple actions, but they do not cause chemotaxis. Complement C3a causes increased vascular permeability by releasing histamine from mast cells.

10 An experiment is conducted involving cellular aspects of wound healing. Components of the extracellular matrix are analyzed to determine their sites of production and their binding patterns to other tissue components. Which of the following molecules synthesized by fibroblasts can best bind to cellular integrins and extracellular collagen and attach epidermal basal cells to basement membrane? □ (A) Heparin □ (B) Dermatan sulfate □ (C) Procollagen □ (D) Fibronectin □ (E) Hyaluronic acid

1 0 (D) Fibronectin is a key component of the extracellular matrix and has a structure that looks like a paper clip. Fibronectin can be synthesized by monocytes, fibroblasts, and endothelium. Heparin that is infused has an anticoagulant function. Dermatan sulfate, a glycosaminoglycan, acts to form a gel that provides resilience and lubrication. Procollagen produced by fibroblasts is formed into ropelike strands of collagen, which provide tensile strength. Hyaluronic acid binds water to form a gelatinous extracellular matrix.

11 An 18-year-old man lacerated his left hand and required sutures. The sutures were removed 1 week later. Wound healing continued, but the site became disfigured by a prominent raised, nodular scar that developed over the next 2 months. Which of the following terms best describes the process that occurred during this 2-month period? □ (A) Organization □ (B) Dehiscence □ (C) Resolution □ (D) Keloid formation □ (E) Secondary union

1 1 (D) The healing process sometimes results in an exuberant production of collagen, giving rise to a keloid. This tendency may run in families. Organization occurs as granulation tissue is replaced by fibrous tissue. Dehiscence occurs when a wound pulls apart. If normal tissue architecture is restored, resolution of inflammation has occurred. Secondary union describes the process by which large wounds fill in and contract.

13 A 3-year-old boy and other male relatives have a history of multiple recurrent infections, including Aspergillus, Staphylococcus, Serratia, Nocardia, and Pseudomonas species. Physical examination shows generalized tender lymphadenopathy. Laboratory findings show normal numbers of morphologically normal circulating WBCs. This child's increased susceptibility to infection is most likely caused by a defect in which of the following steps of the inflammatory response? □ (A) Activation of macrophages by interferon-γ □ (B) Oxygen-dependent killing of bacteria by neutrophils □ (C) Firm adhesion between leukocytes and endothelial cells □ (D) Synthesis of lysozyme in neutrophil granules □ (E) Opsonization of bacteria by immunoglobulins

1 3 (B) Chronic granulomatous disease is characterized by reduced killing of ingested microbes because of inherited defects in the NADPH oxidase system. Two thirds of cases are X-linked, and one third are autosomal recessive. This system generates superoxide anions (O2 −), essential for the subsequent production of microbicidal products such as H2O2, OH, and HOCl−. Macrophage activation by interferon-γ is a key feature of granulomatous inflammation, which is typical of mycobacterial infections. Firm adhesions between leukocytes and endothelium are impaired in leukocyte adhesion deficiency type 1, in which there is a mutation in the β chain of integrins. Lysozyme contained in neutrophil granules is responsible for oxygen-independent killing of bacteria. Impaired opsonization can lead to infections in states of immunoglobulin deficiency.

15 After the birth of her first child, a 19-year-old woman breastfed the infant for about 1 year. Which of the following processes that occurred in the breast during pregnancy allowed her to breastfeed the infant? □ (A) Stromal hypertrophy □ (B) Lobular hyperplasia □ (C) Epithelial dysplasia □ (D) Intracellular accumulation of fat □ (E) Ductal epithelial metaplasia

1 5 (B) Lobules increase under hormonal influence (mainly progesterone) to provide for normal lactation. The breast stroma plays no role in lactation and may increase with pathologic processes. Epithelial dysplasia denotes disordered growth and maturation of epithelial cells that may progress to cancer. Accumulation of fat within cells is a common manifestation of sublethal cell injury or, uncommonly, of inborn errors in fat metabolism. Epithelial metaplasia in the breast is a pathologic process.

17 A 5-year-old child has a history of recurrent bacterial infections, including pneumonia and otitis media. Analysis of leukocytes collected from the peripheral blood shows a deficiency in myeloperoxidase. Which of the following is the most likely cause of this child's increased susceptibility to infections? □ (A) Defective neutrophil degranulation □ (B) Defective production of prostaglandins □ (C) Failure to produce hydroxy-halide radicals (HOCl−) □ (D) Decreased oxygen consumption after phagocytosis □ (E) Failure to produce hydrogen peroxide

1 7 (C) Myeloperoxidase is present in the azurophilic granules of neutrophils. It converts H2O2 into HOCl−, a powerful oxidant and antimicrobial agent. Degranulation occurs when phagolysosomes are formed with engulfed bacteria in phagocytic vacuoles within the neutrophil cytoplasm. In contrast, prostaglandin production depends on a functioning cyclooxygenase pathway of arachidonic acid metabolism. Oxygen consumption with an oxidative or respiratory burst after phagocytosis is aided by glucose oxidation and activation of neutrophil NADPH oxidase, resulting in generation of superoxide that is converted by spontaneous dismutation to H2O2.

18 In an experiment, various soluble mediators are added to a cell culture containing epidermal cells to determine which of the mediators might be useful for promoting epidermal cell growth. When epidermal growth factor (EGF) is added, it binds to epidermal cell surface receptors, inducing RAS protein activation, with subsequent transcription factor translocation and DNA transcription activation. This effect in the epidermal cells is most likely to be mediated through which of the following intracellular pathways? □ (A) Calcium ion channel □ (B) Cyclic AMP □ (C) Cyclin-dependent kinase □ (D) JAK/STAT system □ (E) Mitogen-activated protein kinase

1 8 (E) The mitogen-activated protein (MAP) kinase cascade is involved in signaling from activation via cell surface receptors for growth factors. This pathway is particularly important for signaling of epidermal growth factor and fibroblast growth factor. Ligand binding, such as occurs with acetylcholine at a nerve-muscle junction, alters the conformation of ion channel receptors to allow flow of specific ions such as calcium into the cell, changing the electric potential across the cell membrane. Cyclic AMP is a "second messenger" that is typically activated via ligand binding to receptors with seven transmembrane segments that associate with GTP-hydrolyzing proteins; chemokine receptors function in this fashion. Cyclin-dependent kinases act within the nucleus. JAK/STAT pathways typically are recruited by cytokine receptors.

10 A 39-year-old woman comes to the physician because she has noticed a lump in her breast. Over the past 2 months, the left breast has become slightly enlarged compared with the right breast. On physical examination, the skin overlying the left breast is thickened, reddish orange, and pitted. Mammography shows a 3-cm underlying density, and a fine-needle aspirate of the density indicates carcinoma. Which of the following mechanisms best explains the gross appearance of the left breast? □ (A) Venous thrombosis □ (B) Lymphatic obstruction □ (C) Ischemia □ (D) Chronic passive congestion □ (E) Chronic inflammation

10 (B) Spread of the cancer to the dermal lymphatics produces a peau d'orange appearance of the breast. Because the breast has an extensive venous drainage, cancer or other focal mass lesions are unlikely to cause significant congestion and edema of the breast. Ischemia is rare in the breast because of the abundant arterial supply. Passive congestion does not involve the breast. Chronic inflammation is rare in breast tissue and is not associated with cancer.

10 For the past 2 days, a 41-year-old man has had a severe headache, and he now has a temperature of 39.2°C. A lumbar puncture is performed, and the cerebrospinal fluid obtained has a WBC count of 910/mm3 with 94% neutrophils and 6% lymphocytes. Which of the following substances is the most likely mediator for the fever observed in this patient? □ (A) Bradykinin □ (B) Leukotriene B4 □ (C) Histamine □ (D) Myeloperoxidase □ (E) Nitric oxide □ (F) Phospholipase C □ (G) Tumor necrosis factor

10 (G) Fever is produced by various inflammatory mediators, but the major cytokines that produce fever are interleukin-1 (IL-1) and tumor necrosis factor (TNF), which are produced by macrophages and other cell types. IL-1 and TNF can have autocrine, paracrine, and endocrine effects. They mediate the acute phase responses, such as fever, nausea, and neutrophil release from marrow. Bradykinin, generated from the kinin system on surface contact of Hageman factor with collagen and basement membrane from vascular injury, promotes vascular permeability, smooth muscle contraction, and pain. Leukotriene B4, generated in the lipoxygenase pathway of arachidonic acid metabolism, is a potent neutrophil chemotactic factor. Histamine released from mast cells is a potent vasodilator, increasing vascular permeability. Myeloperoxidase is contained within the azurophilic granules of neutrophils and in the presence of halide converts hydrogen peroxide to HOCl−, which destroys phagocytized organisms by halogenation. Nitric oxide generated by macrophages aids in destruction of microorganisms; nitric oxide released from endothelium mediates vasodilation and inhibits platelet activation. Phospholipase C, which catalyzes the release of arachidonic acid, is generated from platelet activation.

11 A 6-year-old child has a history of recurrent infections with pyogenic bacteria, including Staphylococcus aureus and Streptococcus pneumoniae. The infections are accompanied by a neutrophilic leukocytosis. Microscopic examination of a biopsy specimen obtained from an area of soft tissue necrosis shows microbial organisms, but very few neutrophils. An analysis of neutrophil function shows a defect in rolling. This child's increased susceptibility to infection is most likely caused by a defect in which of the following molecules? □ (A) Selectins □ (B) Integrins □ (C) Leukotriene B4 □ (D) Complement C3b □ (E) NADPH oxidase

11 (A) The patient has a defect in leukocyte rolling, the first step in transmigration of neutrophils from the vasculature to the tissues. Rolling depends on interaction between selectins (P-selectin and E-selectin on endothelial cells, and LRobbins & Cotran Review of Pathology Pg. 26 selectin on neutrophils) and their sialylated ligand molecules (e.g., sialylated Lewis X). Integrins are involved in the next step of transmigration, during which there is firm adhesion between neutrophils and endothelial cells. Leukotriene B4 is a chemotactic agent, complement C3b facilitates phagocytosis, and NADPH oxidase is involved in microbicidal activity.

11 An experiment analyzes cells for enzyme activity associated with sustained cellular proliferation. Which of the following cells is most likely to have the highest telomerase activity? □ (A) Endothelial cells □ (B) Germ cells □ (C) Neurons □ (D) Neutrophils □ (E) Erythrocytes

11 (B) Germ cells have the highest telomerase activity, and the telomere length can be stabilized in these cells. This allows testicular germ cells to retain the ability to divide throughout life. Normal somatic cells have no telomerase activity, and telomeres progressively shorten with each cell division until growth arrest occurs.

11 A 29-year-old woman has a history of frequent nosebleeds and increased menstrual flow. On physical examination, petechiae and purpura are present on the skin of her extremities. Laboratory studies show normal partial thromboplastin time, prothrombin time, and platelet count, and decreased von Willebrand factor activity. This patient most likely has a derangement in which of the following steps in hemostasis? □ (A) Vasoconstriction □ (B) Platelet adhesion □ (C) Platelet aggregation □ (D) Prothrombin generation □ (E) Prothrombin inhibition □ (F) Fibrin polymerization

11 (B) Von Willebrand's factor acts as a "glue" between platelets and the exposed extracellular matrix of the vessel wall after vascular injury. None of the other steps listed depends on von Willebrand's factor. Because the patient's prothrombin time is normal, a lack of prothrombin or the presence of an inhibitor is unlikely.

12 An experiment involves factors controlling wound healing. Skin ulcerations are observed, and the factors involved in the healing process are analyzed. Which of the following factors is most likely to be effective in promoting angiogenesis? □ (A) Platelet-derived growth factor □ (B) Epidermal growth factor □ (C) Basic fibroblast growth factor □ (D) Endostatin □ (E) Interleukin-1

12 (C) Basic fibroblast growth factor is a potent inducer of angiogenesis. It can participate in all steps of angiogenesis. Epidermal growth factor and interleukin-1 have no angiogenic activity. Platelet-derived growth factor plays a role in vascular remodeling. Endostatin is an inhibitor of angiogenesis.

12 A 32-year-old man experiences "heartburn" and gastric reflux after eating a large meal. After many months of symptoms, he undergoes upper gastrointestinal endoscopy, and a biopsy specimen of the esophageal epithelium is obtained. Which of the following pathologic changes, seen in the figure, has most likely occurred? □ (A) Squamous metaplasia □ (B) Mucosal hypertrophy □ (C) Columnar epithelial metaplasia □ (D) Atrophy of lamina propria □ (E) Goblet cell hyperplasia

12 (C) Inflammation from reflux of gastric acid has resulted in replacement of normal esophageal squamous epithelium by intestinal-type columnar epithelium with goblet cells. Such conversion of one adult cell type to another cell type is called metaplasia. The cells are not significantly increased in size (hypertrophic). The lamina propria has some inflammatory cells, but is not atrophic. Goblet cells are not normal constituents of the esophageal mucosa.

12 One month after an appendectomy, a 25-year-old woman palpates a small nodule beneath the skin at the site of the healed right lower quadrant incision. The nodule is excised, and microscopic examination shows macrophages, collagen, a few small lymphocytes, and multinucleated giant cells. Polarizable, refractile material is seen in the nodule. Which of the following complications of the surgery best accounts for these findings? □ (A) Chronic inflammation □ (B) Abscess formation □ (C) Suture granuloma □ (D) Ulceration □ (E) Edema

12 (C) The polarizable material is the suture, and a multinucleated giant cell reaction, typically with foreign body giant cells, is characteristic of a granulomatous reaction to foreign material. Chronic inflammation alone is unlikely to produce a localized nodule with giant cells. An abscess, typically from a wound infection, would have liquefactive necrosis and numerous neutrophils. An ulceration involves loss of epidermis or other epithelial layer. Edema refers to accumulation of fluid in the interstitial space. It does not produce a cellular nodule.

13 On day 28 of her menstrual cycle, a 23-year-old woman experiences onset of menstrual bleeding that lasts for 6 days. She has had regular cycles for many years. Which of the following processes is most likely occurring in the endometrium just before the onset of bleeding? □ (A) Apoptosis □ (B) Caseous necrosis □ (C) Heterophagocytosis □ (D) Atrophy □ (E) Liquefactive necrosis

13 (A) The onset of menstruation is an example of orderly, programmed cell death (apoptosis) through hormonal stimuli. The endometrium breaks down, sloughs off, and then regenerates. Caseous necrosis is typical of granulomatous inflammation, resulting most commonly from mycobacterial infection. Heterophagocytosis is typified by the clearing of an area of necrosis through macrophage ingestion of the necrotic cells. With cellular atrophy, there is often no visible necrosis, but the tissues shrink, something that occurs in the endometrium after menopause. Liquefactive necrosis can occur in any tissue after acute bacterial infection or in the brain after ischemia.

13 A 24-year-old man with acute appendicitis undergoes surgical removal of the inflamed appendix. The incision site is sutured. A trichrome-stained section of the site is shown in the figure. How long after the surgery would this appearance most likely be seen? □ (A) 1 day □ (B) 2 to 3 days □ (C) 4 to 5 days □ (D) 2 weeks □ (E) 1 month

13 (E) The figure shows dense collagen with some remaining dilated blood vessels, typical of the final phase of wound healing, which is extensive by the end of the first month. On day 1, the wound is filled only with fibrin and inflammatory cells. Macrophages and granulation tissue are seen 2 to 3 days postoperatively. Neovascularization is most prominent by days 4 and 5. By week 2, collagen is prominent, and fewer vessels and inflammatory cells are seen.

14 A 50-year-old woman tests positive for hepatitis A antibody. The serum AST level is 275 U/L, and ALT is 310 U/L. One month later, these enzyme levels have returned to normal. Which phase of the cell cycle best describes the hepatocytes 1 month after infection? □ (A) G0 □ (B) G1 □ (C) S □ (D) G2 □ (E) M

14 (A) Hepatocytes are quiescent (stable) cells that can reenter the cell cycle and proliferate in response to hepatic injury, enabling the liver to regenerate partially. Acute hepatitis results in hepatocyte necrosis, marked by elevations in AST and ALT. After the acute process has ended, cells return to the G0 phase, and the liver becomes quiescent again.

14 In a clinical trial, a chemotherapeutic agent is given to patients with breast cancer metastases. Samples of the cancer cells are obtained and assessed for the presence of death of tumor cells by apoptosis. Mutational inactivation of which of the following products is most likely to render tumor cells resistant to the effects of such an agent? □ (A) BCL-2 □ (B) p53 □ (C) NF-κB □ (D) Cytochrome P-450 □ (E) Granzyme B

14 (B) When DNA damage is induced by chemotherapeutic drugs (or other agents), normal p53 genes trigger the cells to undergo apoptosis. When p53 is inactivated, this pathway of cell death can be blocked, rendering the chemotherapy less effective. BCL-2 and NF-κB activity favor cell survival. Cytochrome P-450 does not affect apoptosis. Granzyme B can trigger apoptosis, but it is found in cytotoxic T cells and not in tumor cells.

14 In an experiment, neutrophils collected from peripheral blood are analyzed for a "burst" of oxygen consumption. This respiratory burst is an essential step for which of the following events in an acute inflammatory response? □ (A) Increased production in bone marrow □ (B) Attachment to endothelial cells □ (C) Opsonization of bacteria □ (D) Phagocytosis of bacteria □ (E) Generation of microbicidal activity

14 (E) The respiratory, or oxidative, burst generates reactive oxygen species (i.e., superoxide anion) that are important in destruction of engulfed bacteria. Myelopoiesis does not depend on generation of superoxide. Endothelial attachment of neutrophils is aided by adhesion molecules on the endothelium and the neutrophil surface. These molecules include selectins and integrins. Bacteria are opsonized by complement C3b and IgG, allowing the bacteria to be more readily phagocytosed.

15 In an experiment, the role of low-density lipoprotein (LDL) receptors in uptake of lipids in the liver is studied. A mouse model is created in which the LDL receptor gene is not expressed in the liver. For creating such a "knockout" mouse, which of the following cells would be most useful? □ (A) Adult bone marrow mesenchymal progenitor cells □ (B) Embryonic stem cells in culture □ (C) Hepatic oval cells □ (D) Hematopoietic stem cells □ (E) Regenerating hepatocytes

15 (B) Embryonic stem (ES) cells are multipotent and can give rise to all cells, including hepatocytes. Gene targeting to produce "knockout" mice is done in cultures of ES cells, which are then injected into mouse blastocysts and implanted into the uterus of a surrogate mother. Mesenchymal stem cells also are multipotential, but they are not useful for gene targeting. Hematopoietic stem cells can give rise to all hematopoietic cells, but not other types of cells. Hepatocytes and oval cells within the liver can give rise only to liver cells.

9 A 68-year-old woman suddenly lost consciousness; on awakening 1 hour later, she could not speak or move her right arm and leg. Two months later, a head CT scan showed a large cystic area in the left parietal lobe. Which of the following pathologic processes has most likely occurred in the brain? □ (A) Fat necrosis □ (B) Coagulative necrosis □ (C) Apoptosis □ (D) Liquefactive necrosis □ (E) Karyolysis

9 (D) The high lipid content of central nervous system tissues results in liquefactive necrosis as a consequence of ischemic injury, as in this case of a "stroke." Fat necrosis is seen in breast and pancreatic tissues. Coagulative necrosis is the typical result of ischemia in most solid organs. Apoptosis affects single cells and typically is not grossly visible. Karyolysis refers to fading away of cell nuclei in dead cells.

15 A 20-year-old, sexually active woman experiences lower abdominal pain of 24 hours' duration. She has no previous history of this type of pain. Her temperature is 37.9°C, and on palpation, the left lower abdomen is markedly tender. Laboratory findings include a total WBC count of 29,000/mm3 with 75% segmented neutrophils, 6% bands, 14% lymphocytes, and 5% monocytes. Laparotomy reveals a distended, fluid-filled, reddened left fallopian tube that is about to rupture. A left salpingectomy is performed. Which of the following is most likely to be seen on microscopic examination of the excised fallopian tube? □ (A) Fibroblastic proliferation □ (B) Langhans giant cells □ (C) Liquefactive necrosis □ (D) Mononuclear infiltrates □ (E) Squamous metaplasia

15 (C) This patient is experiencing an acute inflammatory response, with edema, erythema, and pain of short duration. Neutrophils form an exudate and release various proteases, which can produce liquefactive necrosis, starting at the mucosa and extending through the wall of the tube. This mechanism results in perforation. Fibroblasts are more likely participants in chronic inflammatory responses and in healing responses, generally appearing more than 1 week after the initial event. Langhans giant cells are a feature of granulomatous inflammation. Mononuclear infiltrates are more typical of chronic inflammation of the fallopian tube, in which rupture is less likely. Epithelial metaplasia is most likely to occur in the setting of chronic irritation with inflammation.

16 A 9-year-old boy has had a chronic cough and fever for the past month. A chest radiograph shows enlargement of hilar lymph nodes and bilateral pulmonary nodular interstitial infiltrates. A sputum sample contains acid-fast bacilli. A transbronchial biopsy specimen shows granulomatous inflammation within the lung, marked by the presence of Langhans giant cells. Which of the following mediators is most likely to contribute to giant cell formation? □ (A) Tumor necrosis factor □ (B) Complement C3b □ (C) Leukotriene B4 □ (D) Interferon-γ □ (E) Interleukin-1

16 (D) Interferon-γ is secreted by activated T cells and is an important mediator of granulomatous inflammation. It causes activation of macrophages and their transformation into epithelioid cells and then giant cells. Tumor necrosis factor can be secreted by activated macrophages and induces activation of lymphocytes and proliferation of fibroblasts, which are other elements of a granuloma. Complement C3b acts as an opsonin in acute inflammatory reactions. Leukotriene B4 induces chemotaxis in acute inflammatory processes. Interleukin-1 can be secreted by macrophages to produce various effects, including fever, leukocyte adherence, fibroblast proliferation, and cytokine secretion.

16 A 22-year-old woman has a congenital anemia that has required multiple transfusions of RBCs for many years. On physical examination, she now has no significant findings; however, liver function tests show reduced serum albumin. Which of the following findings would most likely appear in a liver biopsy specimen? □ (A) Steatosis in hepatocytes □ (B) Bilirubin in canaliculi □ (C) Glycogen in hepatocytes □ (D) Amyloid in portal triads □ (E) Hemosiderin in hepatocytes

16 (E) Each unit of blood contains about 250 mg of iron. The body has no mechanism for getting rid of excess iron. About 10 to 20 mg of iron per day is lost with normal desquamation of epithelia; menstruating women lose slightly more. Any excess iron becomes storage iron, or hemosiderin. Over time, hemosiderosis involves more and more tissues of the body, particularly the liver. Initially, hemosiderin deposits are found in Kupffer cells and other mononuclear phagocytes in the bone marrow, spleen, and lymph nodes. With great excess of iron, liver cells also accumulate iron. Steatosis usually Robbins & Cotran Review of Pathology Pg. 12 occurs with ingestion of hepatotoxins, such as alcohol. Bilirubin, a breakdown product of blood, can be excreted in the bile so that a person does not become jaundiced. Glycogen storage diseases are inherited and present in childhood. Amyloid is an abnormal protein derived from a variety of precursors, such as immunoglobulin light chains.

16 In an experiment, glass beads are embolized into the coronary arteries of rats, resulting in myocardial injury. After 7 days, sections of the myocardium are studied using light microscopy. The microscopic appearance of one of these sections is shown in the figure. Which of the following mediators is most likely being expressed to produce this appearance? □ (A) Epidermal growth factor □ (B) Interleukin-2 □ (C) Leukotriene B4 □ (D) Thromboxane A2 □ (E) Vascular endothelial growth factor □ (F) Tumor necrosis factor

16 (E) The figure shows a subacute infarction with granulation tissue formation containing numerous capillaries stimulated by vascular endothelial growth factor, representing a healing response. Epidermal growth factor aids in re-epithelialization of a surface wound. Interleukin-2 mediates lymphocyte activation. Leukotriene B4 mediates vasoconstriction and bronchoconstriction. Thromboxane A2 aids vasoconstriction and platelet aggregation. Tumor necrosis factor induces endothelial activation and many responses that occur secondary to inflammation, including fever, loss of appetite, sleep disturbances, hypotension, and increased corticosteroid production.

17 A 50-year-old man experienced an episode of chest pain 6 hours before his death. A histologic section of left ventricular myocardium taken at autopsy showed a deeply eosinophilic-staining area with loss of nuclei and cross-striations in myocardial fibers. There was no hemorrhage or inflammation. Which of the following conditions most likely produced these myocardial changes? □ (A) Viral infection □ (B) Coronary artery thrombosis □ (C) Blunt chest trauma □ (D) Antibodies directed against myocardium □ (E) Protein-deficient diet

17 (B) The deep eosinophilic staining, loss of nuclei, and loss of cell structure suggest an early ischemic injury, resulting in coagulative necrosis. This finding is typically caused by loss of blood flow. Viral infection could cause necrosis of the myocardium, but this is usually accompanied by an inflammatory infiltrate consisting of lymphocytes and macrophages. Blunt trauma produces hemorrhage. An immunological injury may produce focal cell injury, but not widespread ischemic injury. Lack of protein leads to a catabolic state with gradual decrease in cell size, but it does not cause ischemic changes.

17 An experiment is conducted in which the time sequence of events in wound healing is analyzed. Histologic sections are produced from samples of the tissue at the site of a small superficial skin incision in laboratory animals. During the first week, the number of macrophages that are activated to phagocytize tissue debris increases. Which of the following signaling molecules is most likely to play a significant role in producing this finding? □ (A) Acetylcholine □ (B) Cyclic AMP □ (C) Heparan sulfate □ (D) Interferon-γ □ (E) Transforming growth factor β

17 (D) Interferon-γ is a cytokine secreted by lymphocytes at the site of inflammation; it has a paracrine effect that causes transformation of monocytes to activated macrophages. Acetylcholine is a neurotransmitter that does not act on inflammatory cells. Cyclic AMP acts as a "second messenger" within a cell. Heparan sulfate is a component of the extracellular matrix that does not have a signaling function. Transforming growth factor β has an inhibitory function on leukocytes.

18 A 78-year-old woman experiences a sudden loss of consciousness, with loss of movement on the right side of the body. Cerebral angiography shows an occlusion of the left middle cerebral artery. To prevent further ischemic injury to the cerebral cortex, which of the following mediators would be most beneficial? □ (A) Thromboxane A2 □ (B) Bradykinin □ (C) Nitric oxide □ (D) Platelet-activating factor □ (E) Leukotriene E4

18 (C) Endothelial cells can release nitric oxide to produce vasodilation. Nitric oxide also can be administered to patients to promote vasodilation in areas of ischemic injury. Thromboxane A2, platelet-activating factor, and leukotriene E4 have vasoconstrictive properties. Bradykinin mainly increases vascular permeability and produces pain.

18 A 69-year-old man has had difficulty with urination, including hesitancy and frequency, for the past 5 years. A digital rectal examination reveals that the prostate gland is palpably enlarged to about twice normal size. A transurethral resection of the prostate is performed, and the microscopic appearance of the prostate "chips" obtained is that of nodules of glands with intervening stroma. Which of the following pathologic processes has most likely occurred in the prostate? □ (A) Apoptosis □ (B) Dysplasia □ (C) Fatty change □ (D) Hyperplasia □ (E) Hypertrophy □ (F) Metaplasia

18 (D) Nodular prostatic hyperplasia (also known as benign prostatic hyperplasia [BPH]) is a common condition in older men that results from proliferation of prostatic glands and stroma. The prostate becomes more sensitive to androgenic stimulation with age. This is an example of pathologic hyperplasia. Apoptosis results in a loss of, not an increase in, cells. Dysplasia refers to disordered epithelial cell growth and maturation. Fatty change in hepatocytes may produce hepatomegaly. Although BPH is often called "benign prostatic hypertrophy," this term is technically incorrect; it is the number of glands and stromal cells that is increased, rather than the size of existing cells. A change in the glandular epithelium to squamous epithelium would be an example of metaplasia.

19 A 54-year-old man experienced onset of severe substernal chest pain over 3 hours. An ECG showed changes consistent with an acute myocardial infarction. After thrombolytic therapy with tissue plasminogen activator (t-PA), his serum creatine kinase (CK) level increased. Which of the following events most likely occurred after t-PA therapy? □ (A) Reperfusion injury □ (B) Cellular regeneration □ (C) Chemical injury □ (D) Increased synthesis of CK □ (E) Myofiber atrophy

19 (A) If the existing cell damage is not great after myocardial infarction, the restoration of blood flow can help prevent further damage. The reperfusion of damaged cells results in generation of oxygen-derived free radicals, however, causing a reperfusion injury. The elevation in the CK level is indicative of myocardial cell necrosis because this intracellular enzyme does not leak in large quantities from intact cells. Myocardial fibers do not regenerate to a significant degree, and atrophic fibers would have less enzyme to release. t-PA does not produce a chemical injury; it induces thrombolysis to restore blood flow in blocked coronary arteries.

19 Dermal fibroblasts are harvested from the skin biopsy specimen of an adult man. These fibroblasts are transduced with genes encoding for transcription factors including Sox2 and c-myc. Under appropriate culture conditions these cells are then able to generate endodermal, mesodermal, and ectodermal cells. Into which of the following kinds of stem cell have these fibroblasts been transformed? □ (A) Embryonic □ (B) Lineage-committed □ (C) Mesenchymal □ (D) Pleuripotent

19 (D) These transformed cells are designated iPS cells because they have been induced to become pleuripotent. This transformation process gets around the problem of using embryonic stem cells derived from manipulation of human embryos, which has ethical and religious concerns. Embryonic stem cells are totipotent, but they become pleuripotent cells that can further divide into many different cell lines, yet maintain themselves in a replicating pool. Thus, pleuripotent cells are the next best thing compared to embryonic cells for deriving human cells that could replace damaged or diseased tissues. Further differentiation of pleuripotent cells gives rise to cells with more restricted developmental capacity, such as mesenchymal stem cells that can give rise to tissues such as muscle and cartilage but not endodermal or ectodermal cells.

19 A 50-year-old man has experienced midabdominal pain for several weeks. He is afebrile. There is mild upper abdominal tenderness on palpation, and bowel sounds are present. Stool is positive for occult blood. An upper gastrointestinal endoscopy is performed, and biopsy specimens are taken. The figure shows microscopic examination of a biopsy specimen of a duodenal lesion. Which of the following terms best describes this lesion? □ (A) Abscess □ (B) Caseating granuloma □ (C) Chronic inflammation □ (D) Purulent exudate □ (E) Serous effusion □ (F) Ulceration

19 (F) Inflammation involving an epithelial surface may cause such extensive necrosis that the surface becomes eroded, forming an ulcer. If the inflammation continues, the ulcer can continue to penetrate downward into submucosa and muscularis. Alternatively, the ulcer may heal or may remain chronically inflamed. An abscess is a localized collection of neutrophils in tissues. A caseating granuloma is a granulomatous inflammation with central necrosis; the necrosis has elements of liquefaction and coagulative necrosis. Chronic inflammation occurs when there is a preponderance of mononuclear cells, such as lymphocytes, macrophages, and plasma cells, in a process that has gone on for more than a few days—more likely weeks or months—or that accompanies repeated bouts of acute inflammation. Pus, or a purulent exudate, appears semiliquid and yellowish because of the large numbers of granulocytes present. A serous effusion is a watery-appearing transudate that resembles an ultrafiltrate of blood plasma, with a low cell and protein content.

2 A 60-year-old woman developed chest pain that persisted for 4 hours. A radiographic imaging procedure showed an apparent myocardial infarction involving a 3 × 4 cm area of the posterior left ventricular free wall. Laboratory findings showed serum creatine kinase of 600 U/L. The patient received anti-arrhythmic and pressor agents to treat the decreased cardiac output while in the hospital. Which of the following pathologic findings would most likely be seen in the left ventricle 1 month later? □ (A) Abscess □ (B) Complete resolution □ (C) Coagulative necrosis □ (D) Nodular regeneration □ (E) Fibrous scar

2 (E) The elevated creatine kinase level indicates that myocardial necrosis has occurred. The destruction of myocardial fibers precludes complete resolution. The area of myocardial necrosis is gradually replaced by a fibrous scar. Liquefactive necrosis with abscess formation is not a feature of ischemic myocardial injury. Coagulative necrosis is typical of myocardial infarction, but after 1 month, a scar would be present. Nodular regeneration is typical of hepatocyte injury because hepatocytes are stable cells.

2 A 16-year-old boy sustained blunt trauma to the abdomen when the vehicle he was driving struck a bridge abutment at high speed. Peritoneal lavage shows a hemoperitoneum, and at laparotomy, a small portion of the left lobe of the liver is removed because of the injury. Several weeks later, a CT scan of the abdomen shows that the liver has nearly regained its size before the injury. Which of the following processes best explains this CT scan finding? □ (A) Apoptosis □ (B) Dysplasia □ (C) Fatty change □ (D) Hydropic change □ (E) Hyperplasia □ (F) Hypertrophy □ (G) Metaplasia

2 (E) The liver is one of the few organs in the human body that can partially regenerate. This is a form of compensatory hyperplasia. The stimuli to hepatocyte mitotic activity cease when the liver has attained its normal size. Apoptosis is single cell death and frequently occurs with viral hepatitis. Dysplasia is disordered epithelial cell growth that can be premalignant. Fatty change can lead to hepatomegaly; this is not a regenerative process, but is the result of toxic hepatocyte injury. Hydropic change, or cell swelling, does not produce regeneration. Hepatocytes can reenter the cell cycle and proliferate to regenerate the liver; they do not just increase in size.

2 A 73-year-old man was diagnosed 1 year ago with pancreatic adenocarcinoma. He now sees his physician because of a transient ischemic attack. On auscultation of the chest, a heart murmur is heard. Echocardiography shows a 1-cm nodular lesion on the superior aspect of the anterior mitral valve leaflet. The valve leaflet appears to be intact. The blood culture is negative. Which of the following terms best describes this mitral valve lesion? □ (A) Adenocarcinoma □ (B) Atheroma □ (C) Chronic passive congestion □ (D) Mural thrombus □ (E) Petechial hemorrhage □ (F) Phlebothrombosis □ (G) Vegetation

2 (G) A thrombotic mass that forms on a cardiac valve (or, less commonly, on the cardiac mural endocardium) is known as a vegetation. Such vegetations may produce thromboemboli. Vegetations on the right-sided heart valves may embolize to the lungs; vegetations on the left embolize systemically to organs such as the brain, spleen, and kidney. A so-called paradoxical embolus occurs when a right-sided cardiac thrombus crosses a patent foramen ovale and enters the systemic arterial circulation. Patients with cancer may have a hypercoagulable state (e.g., Trousseau syndrome, with malignant neoplasms) that favors the development of arterial and venous thromboses. An adenocarcinoma is a malignant neoplasm that arises from glandular epithelium, forming a mass lesion; endocardial metastases are quite rare. Atheromas form in arteries and do not typically involve the cardiac valves. Chronic passive congestion refers to capillary, sinusoidal, or venous stasis of blood within an organ such as the lungs or liver. Mural thrombi are thrombi that form on the surfaces of the heart or large arteries. The term typically is reserved for large thrombi in a cardiac chamber or dilated aorta or large aortic branch; it is not used to describe thrombotic lesions on cardiac valves. A petechial hemorrhage is a grossly pinpoint hemorrhage. Phlebothrombosis occurs when stasis in large veins promotes thrombosis formation.

22 A 70-year-old man died suddenly. At autopsy, multiple tissue sites were sampled for microscopic analysis. Examination of the tissues showed noncrystalline amorphous deposits of calcium salts in gastric mucosa, renal interstitium, and alveolar walls of lungs. Which of the following conditions would most likely explain these findings? □ (A) Chronic hepatitis □ (B) Chronic glomerulonephritis □ (C) Disseminated tuberculosis □ (D) Generalized atherosclerosis □ (E) Normal aging process □ (F) Pulmonary emphysema

2 2 (B) The microscopic findings suggest metastatic calcification, with deposition of calcium salts in tissues that have physiologic mechanisms for losing acid, creating an internal alkaline environment that favors calcium precipitation. Hypercalcemia can have a variety of causes, including hyperparathyroidism, bone destruction secondary to metastases, paraneoplastic syndromes, and, less commonly, vitamin D toxicity or sarcoidosis. Chronic renal disease reduces phosphate excretion by the kidney, resulting in an increase in serum phosphate. Because the solubility product of calcium and phosphorus must be maintained, the serum calcium is depressed, triggering increased parathyroid hormone output to increase the calcium level, which promotes calcium deposition. Chronic hepatitis leads to hyperbilirubinemia and jaundice. The granulomas of tuberculosis have caseous necrosis with dystrophic calcification. Another form of dystrophic calcification occurs when atherosclerotic lesions calcify. Dystrophic calcification is seen more often in the elderly, but it is the result of a lifetime of pathologic changes, not aging itself. Pulmonary emphysema can lead to respiratory acidosis that is compensated by metabolic alkalosis, with the result that the serum calcium level remains relatively unchanged.

25 A 30-year-old woman with a history of a congenital ventricular septal defect has had a persistent temperature of 38.6°C and headache for the past 3 weeks. A head CT scan shows an enhancing 3-cm, ring-like lesion in the right parietal lobe. Which of the following actions by inflammatory cells most likely produces this CT finding? □ (A) Formation of nitric oxide by macrophages □ (B) Production of interferon-γ by lymphocytes □ (C) Formation of transforming growth factor-β by macrophages □ (D) Generation of prostaglandin by endothelium □ (E) Release of lysosomal enzymes from neutrophils

2 5 (E) This patient has infective endocarditis with septic embolization, producing a cerebral abscess. The tissue destruction that accompanies abscess formation as part of acute inflammatory processes occurs from lysosomal enzymatic destruction, aided by release of reactive oxygen species. Nitric oxide generated by macrophages aids in destruction of infectious agents. Interferon-γ released from lymphocytes plays a major role in chronic and granulomatous inflammatory responses. Transforming growth factor-β formed by macrophages promotes fibrosis. Prostaglandins produced by endothelium promote vasodilation.

26 At autopsy, the heart of a 63-year-old man weighs only 250 g and has small right and left ventricles. The myocardium is firm, with a dark chocolate-brown color throughout. The coronary arteries show minimal atherosclerotic changes. An excessive amount of which of the following substances would most likely be found in the myocardial fibers of this heart? □ (A) Melanin □ (B) Hemosiderin □ (C) Glycogen □ (D) Lipofuscin □ (E) Bilirubin

2 6 (D) Lipofuscin is a "wear-and-tear" pigment that increases with aging, particularly in liver and myocardium. The pigment has minimal effect on cellular function in most cases. Rarely, there is marked lipofuscin deposition in a small heart, a so-called brown atrophy. Melanin pigment is responsible for skin tone: the more melanin, the darker the skin. Hemosiderin is the breakdown product of hemoglobin that contains the iron. Hearts with excessive iron deposition tend to be large. Glycogen is increased in some inherited enzyme disorders, and when the heart is involved, heart size increases. Bilirubin, another breakdown product of hemoglobin, imparts a yellow appearance (icterus) to tissues.

29 A 70-year-old woman has had worsening shortness of breath for the past week. On physical examination, her temperature is 38.3°C. On percussion, there is dullness over the left lung fields. Thoracentesis yields 800 mL of cloudy yellow fluid from the left pleural cavity. Analysis of the fluid reveals a WBC count of 2500/mm3 with 98% neutrophils and 2% lymphocytes. A Gram stain of the fluid shows gram-positive cocci in clusters. Which of the following terms best describes the process occurring in the left pleural cavity? □ (A) Abscess □ (B) Chronic inflammation □ (C) Edema □ (D) Fibrinous inflammation □ (E) Purulent exudate □ (F) Serous effusion

2 9 (E) Bacterial infections often evoke an acute inflammatory response dominated by neutrophils. The extravasated neutrophils attempt to phagocytose and kill the bacteria. In the process, some neutrophils die, and the release of their lysosomal enzymes can cause liquefactive necrosis of the tissue. This liquefied tissue debris and the live and dead neutrophils comprise pus, or purulent exudate. Such an exudate is typical of bacterial infections that involve body cavities. Another term for purulent exudate in the pleural space is empyema. An abscess is a localized collection of neutrophils within tissues. Chronic inflammation occurs when there is a preponderance of mononuclear cells, such as lymphocytes, macrophages, and plasma cells, in a process that has gone on for more than a few days—more likely weeks or months— or that accompanies repeated bouts of acute inflammation. Edema refers to increased fluid collection within tissues, leading to tissue swelling. In fibrinous inflammation, exudation of blood proteins (including fibrinogen, which polymerizes to fibrin) gives a grossly shaggy appearance to surfaces overlying the inflammation. A serous effusion is a watery-appearing transudate that resembles an ultrafiltrate of blood plasma, with a low cell and protein content.

20 A 72-year-old man with severe emphysema has had worsening right ventricular failure for the past 5 years. For the past 4 days, he has had fever and increasing dyspnea. A chest radiograph shows an accumulation of fluid in the pleural spaces. Fluid obtained by thoracentesis has a specific gravity of 1.030 and contains degenerating neutrophils. The most likely cause of this fluid accumulation is an increase in which of the following mechanisms? □ (A) Colloid osmotic pressure □ (B) Lymphatic pressure □ (C) Vascular permeability □ (D) Renal retention of sodium and water □ (E) Leukocytic diapedesis

20 (C) The formation of an exudate containing a significant amount of protein and cells depends on the "leakiness" of blood vessels, principally venules. The extravascular colloid osmotic pressure increases when exudation has occurred, and the protein content of the extravascular space increases, causing extracellular fluid accumulation. The lymphatics scavenge exuded fluid with protein and reduce the amount of extravascular and extracellular fluid. Sodium and water retention helps drive transudation of fluid. Leukocytosis alone is insufficient for exudation because the leukocytes must be driven to emigrate from the vessels by chemotactic factors.

20 A 33-year-old woman has had increasing lethargy and decreased urine output for the past week. Laboratory studies show serum creatinine level of 4.3 mg/dL and urea nitrogen level of 40 mg/dL. A renal biopsy is performed, and the specimen is examined using electron microscopy. Which of the following morphologic changes most likely suggests a diagnosis of acute tubular necrosis? □ (A) Mitochondrial swelling □ (B) Plasma membrane blebs □ (C) Chromatin clumping □ (D) Nuclear fragmentation □ (E) Ribosomal disaggregation

20 (D) Loss of the nucleus results in cell death. All other cellular morphologic changes listed represent reversible cellular injury. The plasma membrane and intracellular organelles remain functional unless severe damage causes loss of membrane integrity.

21 A 43-year-old man has had a cough and fever for the past 2 months. A chest radiograph shows bilateral nodular densities, some with calcification, located mainly in the upper lobes of the lungs. A transbronchial lung biopsy is performed, yielding a specimen with the microscopic appearance shown in the figure. Which of the following chemical mediators is most important in the pathogenesis of this lesion? □ (A) Complement C5a □ (B) Interferon-γ □ (C) Bradykinin □ (D) Nitric oxide □ (E) Prostaglandin

21 (B) The figure shows a granuloma with many epithelioid cells and prominent large Langhans giant cells. Macrophage stimulation and transformation to epithelioid cells and giant cells are characteristic of granuloma formation. Interferon-γ promotes the formation of epithelioid cells and giant cells. Complement C5a is chemotactic for neutrophils. Although occasional neutrophils are seen in granulomas, neutrophils do not form a major component of granulomatous inflammation. Bradykinin, released in acute inflammatory responses, results in pain. Macrophages can release nitric oxide to destroy other cells, but nitric oxide does not stimulate macrophages to form a granulomatous response. Prostaglandins are mainly involved in the causation of vasodilation and pain in acute inflammatory responses.

21 A 40-year-old man had undifferentiated carcinoma of the lung. Despite chemotherapy, the man died of widespread metastases. At autopsy, tumors were found in many organs. Histologic examination showed many foci in which individual tumor cells appeared shrunken and deeply eosinophilic. Their nuclei exhibited condensed aggregates of chromatin under the nuclear membrane. The process affecting these shrunken tumor cells was most likely triggered by the release of which of the following substances into the cytosol? □ (A) Lipofuscin □ (B) Cytochrome c □ (C) Catalase □ (D) Phospholipase □ (E) BCL-2

21 (B) This histologic picture is typical of apoptosis produced by chemotherapeutic agents. The release of cytochrome from the mitochondria is a key step in many forms of apoptosis, and it leads to the activation of caspases. BCL-2 is an antiapoptotic protein that prevents cytochrome c release and prevents caspase activation. Lipofuscin is a pigmented residue representing undigested cellular organelles in autophagic vacuoles. Catalase is a scavenger of hydrogen peroxide. Phospholipases are activated during necrosis and cause cell membrane damage.

25 A 22-year-old woman with leukemia undergoes bone marrow transplantation and receives partially mismatched donor marrow. One month later, she has a scaling skin rash. Examination of a skin biopsy specimen reveals the cellular change shown in the figure. This change most likely results from which of the following biochemical reactions? □ (A) Activation of caspases □ (B) Reduction of ATP synthesis □ (C) Increase in glycolysis □ (D) Activation of lipases □ (E) Lipid peroxidation

25 (A) This cell is shrunken and has been converted into a dense eosinophilic mass. The surrounding cells are normal, and there is no inflammatory reaction. This pattern is typical of apoptosis. Caspase activation is a universal feature of apoptosis, regardless of the initiating cause. Apoptosis induced in recipient cells from donor lymphocytes occurs with graftversus- host disease. Reduced ATP synthesis and increased glycolysis occur when a cell is subjected to anoxia. These changes are reversible. Lipases are activated in enzymatic fat necrosis. Lipid peroxidation occurs when the cell is injured by free radicals.

22 A 43-year-old man with a ventricular septal defect has had a cough and fever for the past 2 days. On examination, he has a temperature of 37.6°C and a cardiac murmur. A blood culture grows Streptococcus, viridans group. His erythrocyte sedimentation rate (ESR) is increased. Microbial cells are opsonized and cleared. Which of the following chemical mediators is most important in producing these findings? □ (A) Bradykinin □ (B) C-reactive protein □ (C) Interferon-γ □ (D) Nitric oxide □ (E) Prostaglandin □ (F) Tumor necrosis factor

22 (B) This acute inflammatory process leads to production of acute-phase reactants, such as C-reactive protein (CRP), fibrinogen, and serum amyloid A (SAA) protein. These proteins, particularly fibrinogen, and immunoglobulins increase red blood cell rouleaux formation to increase sedimentation, which is a nonspecific indicator of inflammation. CRP production is up-regulated by interleukin-6, whereas fibrinogen and SAA are up-regulated mainly by tumor necrosis factor and interleukin-1. Interferon-γ is a potent stimulator of macrophages. Nitric oxide can induce vasodilation or can assist in microbial killing within macrophages. Prostaglandins are vasodilators.

23 A 63-year-old man has a 2-year history of worsening congestive heart failure. An echocardiogram shows mitral stenosis with left atrial dilation. A thrombus is present in the left atrium. One month later, he experiences left flank pain and notes hematuria. Laboratory testing shows elevated serum AST. Which of the following patterns of tissue injury is most likely to be present? □ (A) Liquefactive necrosis □ (B) Caseous necrosis □ (C) Coagulative necrosis □ (D) Fat necrosis □ (E) Gangrenous necrosis

23 (C) Embolization of the thrombus led to blockage of a renal arterial branch, causing an acute renal infarction in this patient. An ischemic injury to most internal organs produces a pattern of cell death called coagulative necrosis. Liquefactive necrosis occurs after ischemic injury to the brain and is the pattern seen with abscess formation. Caseous necrosis can be seen in various forms of granulomatous inflammation, typified by tuberculosis. Fat necrosis is usually seen in pancreatic and breast tissue. Gangrenous necrosis is a form of coagulative necrosis that usually results from ischemia and affects limbs.

23 In an experiment, peripheral blood T lymphocytes are collected and placed in a medium that preserves their function. The lymphocytes are activated by contact with antigen and incubated for several hours. The supernatant fluid is collected and is found to contain a substance that is a major stimulator of monocytes and macrophages. Which of the following substances is most likely to stimulate these cells? □ (A) Leukotriene B4 □ (B) Histamine □ (C) Interferon-γ □ (D) Interleukin-1 □ (E) Nitric oxide □ (F) Phospholipase C □ (G) Tumor necrosis factor

23 (C) Interferon-γ secreted from lymphocytes stimulates monocytes and macrophages, which secrete their own cytokines that further activate lymphocytes. Interferon-γ also is important in transforming macrophages into epithelioid cells in a granulomatous inflammatory response. Leukotriene B4, generated in the lipoxygenase pathway of arachidonic acid metabolism, is a potent neutrophil chemotactic factor. Histamine released from mast cells is a potent vasodilator, increasing vascular permeability. Interleukin-1 and tumor necrosis factor, both produced by activated macrophages, mediate many systemic effects, including fever, metabolic wasting, and hypotension. Nitric oxide generated by macrophages aids in destruction of microorganisms; nitric oxide released from endothelium mediates vasodilation and inhibits platelet activation. Binding of agonists such as epinephrine, collagen, or thrombin to platelet surface receptors activates phospholipase C, which catalyzes the release of arachidonic acid from two of the major membrane phospholipids, phosphatidylinositol and phosphatidylcholine.

24 At autopsy, a 40-year-old man has an enlarged (2200 g) liver with a yellow cut surface. The microscopic appearance of this liver is shown in the figure. Before death, the man's total serum cholesterol and triglyceride levels were normal, but he had a decreased serum albumin concentration and increased prothrombin time. Which of the following activities most likely led to these findings? □ (A) Injecting heroin □ (B) Playing basketball □ (C) Drinking beer □ (D) Smoking cigarettes □ (E) Ingesting aspirin

24 (C) The appearance of lipid vacuoles in many of the hepatocytes is characteristic of fatty change (steatosis) of the liver. Abnormalities in lipoprotein metabolism can lead to steatosis. Alcohol is a hepatotoxin that produces hepatic steatosis. Decreased serum albumin levels and increased prothrombin time suggest alcohol-induced hepatocyte damage. Substance abuse with heroin produces few organ-specific pathologic findings. Exercise has little direct effect on hepatic function. Smoking directly damages lung tissue, but has no direct effect on the liver. Aspirin has a significant effect on platelet function, but not on hepatocytes.

24 A 90-year-old woman is diagnosed with Staphylococcus aureus pneumonia and receives a course of antibiotic therapy. Two weeks later, she no longer has a productive cough, but she still has a temperature of 38.1°C. A chest radiograph shows a 3-cm rounded density in the right lower lobe of the lung whose liquefied contents form a central air-fluid level. There are no surrounding infiltrates. Which of the following terms best describes the outcome of the patient's pneumonia? □ (A) Complete resolution □ (B) Regeneration □ (C) Fibrosis □ (D) Abscess formation □ (E) Progression to chronic inflammation

24 (D) The formation of a fluid-filled cavity after infection with Staphylococcus aureus suggests that liquefactive necrosis has occurred. The cavity is filled with tissue debris and viable and dead neutrophils (pus). Localized, pus-filled cavities are Robbins & Cotran Review of Pathology Pg. 28 called abscesses. Some bacterial organisms, such as S. aureus, are more likely to be pyogenic, or pus-forming. With complete resolution, the structure of the lung remains almost unaltered. Lung tissue, in contrast to liver, is incapable of regeneration. Scarring or fibrosis may follow acute inflammation as the damaged tissue is replaced by fibrous connective tissue. Most bacterial pneumonias resolve, and progression to continued chronic inflammation is uncommon.

7 A 66-year-old woman comes to the emergency department 30 minutes after the onset of chest pain that radiates to her neck and left arm. She is diaphoretic and hypotensive; the serum troponin I level is elevated. Thrombolytic therapy is begun. Which of the following drugs is most likely to be administered? □ (A) Tissue plasminogen activator □ (B) Aspirin □ (C) Heparin □ (D) Nitric oxide □ (E) Vitamin K

7 (A) Tissue plasminogen activator is a thrombolytic agent that causes the generation of plasmin, which cleaves fibrin to dissolve clots. Aspirin prevents formation of new thrombi by inhibiting platelet aggregation. Heparin prevents thrombosis by activating antithrombin III. Nitric oxide is a vasodilator. Vitamin K is required for synthesis of certain clotting factors.

26 A 35-year-old man has had increasing dyspnea for the past 24 hours. A chest radiograph shows large, bilateral pleural effusions. Thoracentesis yields 500 mL of slightly cloudy yellow fluid from the right pleural cavity. Cytologic examination of the fluid shows many neutrophils, but no lymphocytes or RBCs. Which of the following mechanisms contributes most to the accumulation of the fluid in the pleural space? □ (A) Arteriolar vasoconstriction □ (B) Neutrophil release of lysosomes □ (C) Endothelial contraction □ (D) Inhibition of platelet adherence □ (E) Lymphatic obstruction

26 (C) Exudation of fluid from venules and capillaries is a key component of the acute inflammatory process. Several mechanisms of increased vascular permeability have been proposed, including formation of interendothelial gaps by contraction of endothelium. This is caused by mediators, such as histamine and leukotrienes. The vessels then become more "leaky," and the fluid leaves the intravascular space to accumulate extravascularly, forming effusions in body cavities or edema in tissues. Arteriolar vasoconstriction is a transient response to injury that diminishes blood loss. After neutrophils reach the site of tissue injury outside of the vascular space, they release lysosomal enzymes. Platelets adhere to damaged endothelium and promote hemostasis. Lymphatic obstruction results in the accumulation of protein-rich lymph and lymphocytes, producing a chylous effusion.

27 A 69-year-old woman has had a chronic cough for the past year. A chest radiograph shows a 6-cm mass in the left lung, and a needle biopsy specimen of the mass shows carcinoma. A pneumonectomy is performed, and examination of the hilar lymph nodes reveals a uniform, dark black cut surface. Which of the following factors most likely accounts for the appearance of the lymph nodes? □ (A) Smoking □ (B) Bleeding disorder □ (C) Liver failure □ (D) Aging □ (E) Metastases

27 (A) Anthracotic pigmentation is common in lung and hilar lymph nodes and occurs when carbon pigment is inhaled from polluted air. The tar in cigarette smoke is a major source of such carbonaceous pigment. Resolution of hemorrhage can produce hemosiderin pigmentation, which imparts a brown color to tissues. Hepatic failure may result in jaundice, characterized by a yellow color. Older individuals generally have more anthracotic pigment, but this is not inevitable with aging—individuals living in rural areas with good environmental air quality have less pigment. Metastases impart a tan-towhite appearance to tissues.

27 A 12-month-old boy with a 6-month history of repeated infections has had a fever and cough for the past 3 days. A Gram stain of sputum shows many gram-positive cocci in chains. CBC shows neutrophilia. Laboratory studies show that the patient's neutrophils phagocytose and kill organisms normally in the presence of normal human serum, but not in his own serum. The neutrophils migrate normally in a chemotaxis assay. Which of the following is the most likely cause of the child's increased susceptibility to infection? □ (A) Deficiency of integrins □ (B) Neutrophil microtubular protein defect □ (C) Immunoglobulin deficiency □ (D) Defective neutrophil generation of hydrogen peroxide □ (E) Deficiency of selectins

27 (C) The patient has immunoglobulin deficiency, which prevents opsonization and phagocytosis of microbes. Deficiency of integrins and selectins, or a defect in microtubules, would prevent adhesion and locomotion of neutrophils. H2O2 production is part of the oxygen-dependent killing mechanism. This mechanism is intact in this patient because the neutrophils are able to kill bacteria when immunoglobulins in normal serum allow phagocytosis.

28 A 35-year-old woman takes acetylsalicylic acid (aspirin) for arthritis. Although her joint pain is reduced with this therapy, the inflammatory process continues. The aspirin therapy alleviates her pain mainly through reduction in the synthesis of which of the following mediators? □ (A) Complement C1q □ (B) Prostaglandins □ (C) Leukotriene E4 □ (D) Histamine □ (E) Nitric oxide

28 (B) Prostaglandins are produced through the cyclooxygenase pathway of arachidonic acid metabolism. Aspirin and other nonsteroidal anti-inflammatory drugs block the synthesis of prostaglandins, which can produce pain. Complement C1q is generated in the initial stage of complement activation, which can eventually result in cell lysis. Leukotrienes are generated by the lipoxygenase pathway, which is not blocked by aspirin. Histamine is mainly a vasodilator. Nitric oxide released from endothelium is a vasodilator.

28 A 44-year-old man has a history of poorly controlled diabetes mellitus leading to coronary artery disease. He now has decreasing cardiac output. An increase in which of the following substances in his blood is most indicative of reversible cell injury from decreased perfusion of multiple organs and tissues? □ (A) Carbon dioxide □ (B) Creatinine □ (C) Glucose □ (D) Lactic acid □ (E) Troponin I

28 (D) Decreased tissue perfusion leads to hypoxemia and depletion of ATP when cell metabolism shifts from aerobic to anaerobic glycolysis, resulting in depletion of glycogen stores and increased production and accumulation of lactic acid, reducing intracellular pH. Creatinine would increase with reduced renal function from decreased renal perfusion, but this would not explain the changes in other tissues. An increased glucose would be indicative of poorly controlled diabetes mellitus, not decreased perfusion. Carbon dioxide is likely to be cleared via the lungs, which are still sufficiently perfused. An increase in troponin I suggests irreversible myocardial injury.

29 An experiment introduces a "knockout" gene mutation into a cell line. The frequency of shrunken cells with chromatin clumping and cytoplasmic blebbing is increased compared with a cell line without the mutation. Overall survival of the mutant cell line is reduced. Which of the following genes is most likely to be affected by this mutation? □ (A) BAX □ (B) BCL-2 □ (C) C-MYC □ (D) FAS □ (E) p53

29 (B) These histologic findings are typical of apopotosis. The BCL-2 gene product inhibits cellular apoptosis by binding to Apaf-1. The BAX gene product promotes apoptosis. The C-MYC gene is involved with oncogenesis. The FAS gene encodes for a cellular receptor for FAS ligand, which signals apoptosis. p53 gene activity normally stimulates apoptosis, but mutation favors cell survival.

7 In an experiment, cells are subjected to radiant energy in the form of x-rays. This results in cell injury caused by hydrolysis of water. Which of the following cellular enzymes protects the cells from this type of injury? □ (A) Phospholipase Robbins & Cotran Review of Pathology Pg. 2 □ (A) Phospholipase □ (B) Glutathione peroxidase □ (C) Endonuclease □ (D) Lactate dehydrogenase □ (E) Protease

7 (B) Intracellular mechanisms exist that deal with free radical generation, as can occur with radiant injury from irradiation. Glutathione peroxidase reduces such injury by catalyzing the breakdown of hydrogen peroxide. Phospholipases decrease cellular phospholipids and promote cell membrane injury. Proteases can damage cell membranes and cytoskeletal proteins. Endonucleases damage nuclear chromatin. Lactate dehydrogenase is present in a variety of cells, and its elevation in the serum is an indicator of cell death.

3 A 53-year-old woman has had a high fever and cough productive of yellowish sputum for the past 2 days. Her vital signs include temperature of 37.8°C, pulse of 83/min, respirations of 17/min, and blood pressure of 100/60 mm Hg. On auscultation of the chest, crackles are audible in both lung bases. A chest radiograph shows bilateral patchy pulmonary infiltrates and fluid in the right pleural cavity. Thoracentesis yields 500 mL of cloudy yellow fluid. Which of the following inflammatory cell types is most likely to be seen in greatly increased numbers in a sputum specimen? □ (A) Macrophages □ (B) Neutrophils □ (C) Mast cells □ (D) Small lymphocytes □ (E) Langhans giant cells

3 (B) The patient's signs and symptoms suggest acute bacterial pneumonia. Such infections induce an acute inflammation dominated by neutrophils, which gives the sputum its yellowish, purulent appearance. Macrophages become more numerous after acute events, cleaning up tissue and bacterial debris through phagocytosis. Mast cells are better known as participants in allergic and anaphylactic responses. Lymphocytes are a feature of chronic inflammation. Langhans giant cells are seen with granulomatous inflammatory responses.

3 An experiment infects one group of test animals with viral hepatitis. Two months later, complete recovery of the normal liver architecture is observed when the livers from these animals are examined microscopically. A second test group is infected with bacterial organisms, and after the same period of time, fibrous scars from resolving abscesses are seen microscopically in the livers. Which of the following factors best explains the different outcomes for the two test groups? □ (A) Nature of the etiologic agent □ (B) Extent of liver cell injury □ (C) Injury to the connective tissue framework □ (D) Location of the lesion □ (E) Extent of damage to the bile ducts

3 (C) Hepatocytes are stable cells with an extensive ability to regenerate. The ability to restore normal architecture of an organ such as the liver depends on the viability of the supporting connective tissue framework. If the connective tissue cells are not injured, hepatocyte regeneration can restore normal liver architecture. This regeneration occurs in many cases of viral hepatitis. A liver abscess is associated with liquefactive necrosis of hepatocytes and the supporting connective tissue. It heals by scarring. The other options listed may explain the amount of liver injury, but not the nature of the response.

3 A 21-year-old woman sustains multiple injuries, including fractures of the right femur and tibia and the left humerus, in a motor vehicle collision. She is admitted to the hospital, and the fractures are stabilized surgically. Soon after admission to the hospital, she is in stable condition. She suddenly becomes severely dyspneic, however, 2 days later. Which of the following complications is the most likely cause of this sudden respiratory difficulty? □ (A) Right hemothorax □ (B) Pulmonary edema □ (C) Fat embolism □ (D) Cardiac tamponade □ (E) Pulmonary infarction

3 (C) The mechanism for fat embolism is unknown, in particular, why onset of symptoms is delayed 1 to 3 days after the initial injury (or 1 week for cerebral symptoms). The cumulative effect of many small fat globules filling peripheral pulmonary arteries is the same as one large pulmonary thromboembolus. Hemothorax and cardiac tamponade would be immediate complications after traumatic injury, not delayed events. Pulmonary edema severe enough to cause dyspnea would be unlikely to occur in hospitalized patients because fluid status is closely monitored. Pulmonary infarction may cause dyspnea, but pulmonary thromboembolus from deep venous thrombosis is typically a complication of a longer hospitalization.

3 On a routine visit to the physician, an otherwise healthy 51-year-old man has a blood pressure of 150/95 mm Hg. If his hypertension remains untreated for years, which of the following cellular alterations would most likely be seen in his myocardium? □ (A) Atrophy □ (B) Hyperplasia □ (C) Metaplasia □ (D) Hemosiderosis □ (E) Hypertrophy

3 (E) The pressure load on the left ventricle results in an increase in myofilaments in the existing myofibers. The result of continued stress from hypertension is eventual heart failure with decreased contractility, but the cells do not decrease in size. Metaplasia of muscle does not occur, although loss of muscle occurs with aging as myofibers are replaced by fibrous tissue and adipose tissue. Hemosiderin deposition in the heart is a pathologic process resulting from increased iron stores in the body.

32 The nonpregnant uterus of a 20-year-old woman measures 7 × 4 × 3 cm. The woman becomes pregnant and just before delivery of a term infant, the uterus measures 34 × 18 × 12 cm. Which of the following cellular processes has contributed most to the increase in uterine size? □ (A) Endometrial glandular hyperplasia □ (B) Myometrial fibroblast proliferation □ (C) Endometrial stromal hypertrophy □ (D) Myometrial smooth muscle hypertrophy □ (E) Vascular endothelial hyperplasia

3 2 (D) The increase in uterine size is primarily the result of an increase in myometrial smooth muscle cell size. The endometrium also increases in size, but it remains as a lining to the muscular wall and does not contribute as much to the change in size. There is little stroma in myometrium and a greater proportion in endometrium, but this contributes a smaller percentage to the gain in size than muscle. The vessels are a minor but essential component in this process.

33 A 40-year-old woman has had chronic congestive heart failure for the past 3 years. In the past 2 months, she developed a cough productive of rust-colored sputum. A sputum cytology specimen shows numerous hemosiderin-laden macrophages. Which of the following subcellular structures in macrophages is most important for the accumulation of this pigment? □ (A) Lysosome □ (B) Endoplasmic reticulum □ (C) Ribosome □ (D) Golgi apparatus □ (E) Chromosome

3 3 (A) Heterophagocytosis by macrophages requires that endocytosed vacuoles fuse with lysosomes to degrade the engulfed material. With congestive failure, extravasation of RBCs into alveoli occurs, and pulmonary macrophages must phagocytose the RBCs, breaking down the hemoglobin and recycling the iron by hemosiderin formation.

30 A 5-year-old boy has a history of recurrent infections with gram-positive bacteria, including Staphylococcus aureus. Genetic testing shows a defect leading to a lack of β2-integrin production. Which of the following abnormalities of neutrophil function is most likely responsible for these clinical symptoms? □ (A) Normal neutrophil rolling but inadequate sticking on cytokine-activated endothelial cells □ (B) Failure of neutrophils to migrate to the site of infection after leaving the vasculature □ (C) Reduced respiratory burst in neutrophils after phagocytosis of bacteria □ (D) Diminished phagocytosis of bacteria opsonized with IgG □ (E) Failure to generate hydroxy-halide radicals (HOCl−)

30 (A) During acute inflammation, neutrophils extravasate from the blood vessels. This process depends on adhesion molecules expressed on the neutrophils and endothelial cells. In the first stage of extravasation, the neutrophils "roll over" the endothelium. At this stage, the adhesion between the neutrophils and endothelial cells is weak. Rolling is mediated by binding of selectins to sialylated oligosaccharides. The next step, firm adhesion, is mediated by binding of integrins on the leukocytes to their receptors, intracellular adhesion molecule-1 or vascular cell adhesion molecule-1 (VCAM-1), on endothelial cells. Integrins have two chains, α and β. A genetic lack of β chains prevents firm adhesion of leukocytes to endothelial cells. Neutrophil migration to a site of infection depends on the presence of chemotactic factors that bind to the neutrophil and activate phospholipase C to begin a series of events that culminate in the influx of calcium, which triggers contractile proteins. The respiratory burst to kill phagocytized organisms depends on NADPH oxidase, and a deficiency of this enzyme leads to chronic granulomatous disease. Phagocytosis of opsonized organisms depends on engulfment, which requires contractile proteins in the neutrophil cytoplasm. Formation of HOCl− requires myeloperoxidase released from neutrophil granules.

30 A tissue preparation is experimentally subjected to a hypoxic environment. The cells in this tissue begin to swell, and chromatin begins to clump in the nucleus. ATPases are activated, and ATP production decreases. Which of the following ions released from mitochondria leads to these findings and to eventual cell death? □ (A) Ca2+ □ (B) Cl− □ (C) HCO3 2− □ (D) K+ □ (E) Na+ □ (F) PO4 2+

30 (A) Irreversible cellular injury is likely to occur when calcium increases within cells. This calcium can influx into cells and be released from mitochondria and endoplasmic reticulum. The calcium activates ATPases, phospholipases, proteases, and endonucleases, which injure cell components. Mitochondrial permeability is increased to release cytochrome c, which activates caspases leading to apoptosis. Of the other ions listed, sodium influxes into the cell, while potassium diffuses out when the sodium pump fails as ATP levels fall, but this is potentially reversible.

31 A chest radiograph of an asymptomatic 37-year-old man showed a 3-cm nodule in the middle lobe of the right lung. The nodule was excised with a pulmonary wedge resection, and sectioning showed the nodule to be sharply circumscribed with a soft, white center. Culture of tissue from the nodule grew Mycobacterium tuberculosis. Which of the following pathologic processes has most likely occurred in this nodule? □ (A) Apoptosis □ (B) Caseous necrosis □ (C) Coagulative necrosis □ (D) Fat necrosis □ (E) Fatty change □ (F) Gangrenous necrosis □ (G) Liquefactive necrosis

31 (B) The cheese-like appearance gives this form of necrosis its name—caseous necrosis. In the lung, tuberculosis and fungal infections are most likely to produce this pattern of tissue injury. Apoptosis involves individual cells, without extensive or localized areas of tissue necrosis. Coagulative necrosis is more typical of ischemic tissue injury. Fat necrosis most often occurs in the breast and pancreas. Fatty change is most often a feature of hepatocyte injury, and the cell integrity is maintained. Gangrene characterizes extensive necrosis of multiple cell types in a body region or organ. Liquefactive necrosis is seen in abscesses or ischemic cerebral injury.

31 An experiment isolates peripheral blood cells into a culture medium that preserves their metabolic activity. After interferon-γ is added to this culture, the cells are incubated. Next, a cell-free supernatant from this culture is added to a second culture medium containing Escherichia coli organisms. Which of the following cell types is the most likely source for observed bactericidal activity against E. coli □ (A) Basophil □ (B) B lymphocyte □ (C) CD4 lymphocyte □ (D) CD8 lymphocyte □ (E) Macrophage □ (F) Neutrophil □ (G) Natural killer cell

31 (E) Macrophages contain cytokine-inducible nitric oxide synthase (iNOS), which generates nitric oxide. Nitric oxide, by itself and on interaction with other reactive oxygen species, has antimicrobial activity. CD4 or CD8 lymphocytes can be the source for interferon-γ (IFN-γ), which stimulates macrophage production of NOS. Endothelial cells contain a form of NOS (eNOS) that acts to promote vasodilation. B lymphocytes produce immunoglobulins that can opsonize bacteria. Basophils release histamine and arachidonic acid metabolites, which participate in the acute inflammatory process. Neutrophils can phagocytize microbes, but use NAPDH oxidase and enzymes other than NOS to kill the microbes. Natural killer cells have Fc receptors and can lyse IgG-coated target cells; they also generate IFN-γ.

32 Patients with extensive endothelial injury from Escherichia coli sepsis have consumption of coagulation factors as well as an extensive inflammatory response. Administration of activated protein C decreases this inflammatory response by reducing the amount of a substance that normally binds to protease-activated receptors to trigger expression of adhesion molecules, cytokines, and chemokines. What is this substance most likely to be? □ (A) Complement □ (B) Fibrin □ (C) Kallikrein □ (D) Plasmin □ (E) Thrombin

32 (E) Ongoing activation of coagulation generates an inflammatory response that further amplifies coagulation, creating a vicious cycle. Protein C antagonizes coagulation factor V, which catalyzes activation of prothrombin to thrombin, thereby breaking the cycle of thrombin generation. Complement components can become activated by plasmin (C3) and kallikrein (C5) forming anaphlytoxins (C3a and C5a) that promote inflammation. Fibrin is the end product of coagulation pathways that forms a meshwork entrapping platelets and forming a plug. Kallikrein is generated by activation of Hageman factor (XII) and leads to formation of bradykinin. Plasmin is generated from plasminogen activated by thrombosis to promote clot lysis.

9 A 5-year-old child reaches up to the stove and touches a pot of boiling soup. Within several hours, there is marked erythema of the skin of the fingers on the child's right hand, and small blisters appear on the finger pads. Which of the following terms best describes this process? □ (A) Fibrinous inflammation □ (B) Purulent inflammation □ (C) Serous inflammation □ (D) Ulceration □ (E) Granulomatous inflammation

9 (C) Serous inflammation is the mildest form of acute inflammation. A blister is a good example of serous inflammation. It is associated primarily with exudation of fluid into the subcorneal or subepidermal space. Because the injury is mild, the fluid is relatively protein-poor. A protein-rich exudate results in fibrin accumulation. Acute inflammatory cells, mainly neutrophils, exuded into a body cavity or space form a purulent (suppurative) exudate, typically associated with liquefactive necrosis. Loss of the epithelium leads to ulceration. Granulomatous inflammation is characterized by collections of transformed macrophages called epithelioid cells.

34 In an experiment, a large amount of a drug is administered to subjects and is converted by cytochrome P-450 to a toxic metabolite. The accumulation of this metabolite leads to increased lipid peroxidation within cells, causing damage to cell membranes and cell swelling. Depletion of which of the following substances by this mechanism within the cytosol exacerbates the cellular injury? □ (A) ADP □ (B) Calcium □ (C) Glutathione □ (D) NADPH oxidase □ (E) Nitric oxide synthase □ (F) mRNA □ (G) Sodium

34 (C) Glutathione in the cytosol helps to reduce cellular injury from many toxic metabolites and free radicals. ADP is converted to ATP by oxidative and glycolytic cellular pathways to provide energy that drives cellular functions, and a reduction in ATP leaves the cell vulnerable to injury. Calcium influx into a cell promotes injury. NADPH oxidase generates superoxide, which is used by neutrophils in killing bacteria. Nitric oxide synthase in macrophages produces nitric oxide, which aids in destroying organisms undergoing phagocytosis. Protein synthesis in cells depends on mRNA for longer survival and to recover from damage from free radicals. Failure of the sodium pump leads to increased cytosolic sodium and cell swelling with injury.

35 An experiment is conducted in which cells in tissue culture are subjected to high levels of ultraviolet radiant energy. Electron microscopy shows cellular damage in the form of increased cytosolic aggregates of denatured proteins. In situ hybridization reveals that protein components in these aggregates also are found in proteasomes. Which of the following substances is most likely to bind to the denatured proteins, targeting them for catabolism by cytosolic proteasomes? □ (A) Adenosine monophosphate □ (B) Calcium □ (C) Caspase □ (D) Granzyme B □ (E) Hydrogen peroxide □ (F) Ubiquitin

35 (F) Heat-shock proteins provide for a variety of cellular "housekeeping" activities, including recycling and restoration of damaged proteins and removal of denatured proteins. Ubiquitin targets denatured proteins and facilitates their binding to proteasomes, which then break down the proteins to peptides. ADP increases when ATP is depleted, helping to drive anaerobic glycolysis. Cytosolic calcium levels may increase with cell injury that depletes ATP; the calcium activates phospholipases, endonucleases, and proteases, which damage the cell membranes, structural proteins, and mitochondria. Caspases are enzymes that facilitate apoptosis. Granzyme B is released from cytotoxic T lymphocytes and triggers apoptosis. Hydrogen peroxide is one of the activated oxygen species generated under conditions of cellular ischemia, producing nonspecific damage to cellular structures, particularly membranes.

36 A 71-year-old man diagnosed with pancreatic cancer is noted to have decreasing body mass index. His normal cells comprising skeletal muscle undergo atrophy by sequestering organelles and cytosol in a vacuole followed by fusion with a lysosome. However, the cancer continues to increase in size. Which of the following processes is most likely occurring in the normal cells but inhibited in the cancer cells of this man? □ (A) Aging □ (B) Apoptosis □ (C) Autophagy □ (D) Hyaline change □ (E) Karyorrhexis

36 (C) Autophagy is a form of cellular downsizing in response to stress, as the cell consumes itself, by upregulating Atgs genes. Lipofuscin granules are residual bodies left over from this process. There may be eventual cell death triggered by autophagy, but by a different mechanism than apoptosis, a form of single cell necrosis in which cell fragmentation occurs. Cancer cells acquire the ability to avoid autophagy, perhaps by downregulating PTEN gene expression, and maintain a survival advantage even as the patient is dying. There is slow autophagy with aging, but autophagy is accelerated with stressors such as malnutrition and chronic disease. Hyaline is a generic term for intracellular or extracellular protein accumulations appearing pink and homogenous with H&E staining. Karyorrhexis is nuclear fragmentation in a necrotic cell.

37 A 5-year-old child ingests 50 iron tablets, each with 27 mg of iron. Within 6 hours the child develops abdominal pain and lethargy. On physical examination he is hypotensive. Laboratory studies show metabolic acidosis. Through formation of which of the following compounds is the cell injury in this child most likely mediated? □ (A) Ascorbic acid □ (B) Hemosiderin □ (C) Hydroxyl radical □ (D) Nitric oxide □ (E) Superoxide dismutase

37 (C) Excessive iron ingestion, particularly in a child, can overwhelm the bodys ability to bind the absorbed free iron with the transport protein transferrin. The free iron contributes to generation of free radicals via the Fenton reaction. Ascorbic acid (vitamin C) and vitamin E both act as antioxidants to protect against free radical injury. Hemosiderin is a storage form of iron from excess local or systemic accumulation of ferritin, and by itself does not cause cell injury until large amounts are present, as with hemochromatosis. Nitric oxide generated within macrophages can be utilized to kill microbes. It can be converted to a highly reactive peroxynitrite anion. Superoxide dismutase helps break down superoxide anion to hydrogen peroxide, thus scavenging free radicals.

38 A proponent of Malbec, Syrah, and Merlot wines (all reds) touts their contribution to longevity, but this wine aficionado also controls the caloric content of his diet such that his body mass index is <22. This lifestyle promotes increased insulin sensitivity and glucose utilization. He fully expects to live long because he has read that caloric restriction prolongs life. In this man, which of the following proteins will most likely mediate the effect of calorie restriction upon increased longevity? □ (A) Caspase □ (B) Glutathione □ (C) Sirtuin □ (D) Telomerase □ (E) Ubiquitin

38 (C) The one sure way to increase life span is calorie restriction. But why do without the things we like to do without them longer? Dietary excesses lead to increased morbidity with reduced quality of life, as well as mortality, from chronic diseases such as diabetes mellitus. The histone deacetylase activity of sirtuins may promote transcription of genes encoding for proteins that increase metabolic activity and inhibit effects of free radicals. Red wines have been shown to increase sirtuins, but dont drink too much! Moderation is the key. Glutathione promotes free radical breakdown, though chronic excessive alcohol consumption depletes hepatocyte glutathione. Caspases trigger apoptosis and cell death. Telomerases aid in promoting continued cell division, but cannot be altered by lifestyle, and turning them on is one feature of neoplasia. Ubiquitin is a peptide that is part of the ubiquitin-proteasome pathway of protein degradation seen with nutrient deficiencies, so when you eat less, be sure to eat a balanced diet.

4 A 63-year-old man develops worsening congestive heart failure 2 weeks after an acute myocardial infarction. An echocardiogram shows a markedly decreased ejection fraction. He dies 1 day later. At autopsy, a section of the infarct shows that the necrotic myocardium has largely been replaced by capillaries, fibroblasts, and collagen. Various inflammatory cells are present. Which of the following inflammatory cell types in this lesion plays the most important role in the healing process? □ (A) Macrophages □ (B) Plasma cells □ (C) Neutrophils □ (D) Eosinophils □ (E) Epithelioid cells

4 (A) Macrophages, present in such lesions, play a prominent role in the healing process. Activated macrophages can secrete various cytokines that promote angiogenesis and fibrosis, including platelet-derived growth factor, fibroblast growth factor, interleukin-1, and tumor necrosis factor. Plasma cells can secrete immunoglobulins and are not instrumental to healing of an area of tissue injury. Neutrophils are most numerous within the initial 48 hours after infarction, but are not numerous after the first week. Eosinophils are most prominent in allergic inflammations and in parasitic infections. Epithelioid cells, which are aggregations of activated macrophages, are typically seen with granulomatous inflammation. The healing of acute inflammatory processes does not involve granulomatous inflammation.

4 A 72-year-old man died suddenly from congestive heart failure. At autopsy, the heart weighed 580 g and showed marked left ventricular hypertrophy and minimal coronary arterial atherosclerosis. A serum chemistry panel ordered before death showed no abnormalities. Which of the following pathologic processes best accounts for the appearance of the aortic valve seen in the figure? □ (A) Amyloidosis □ (B) Dystrophic calcification □ (C) Lipofuscin deposition □ (D) Hemosiderosis □ (E) Fatty change

4 (B) The valve is stenotic because of nodular deposits of calcium. The process is "dystrophic" because calcium deposition occurs in damaged tissues. The damage in this patient is a result of the wear and tear of aging. Amyloid deposition in the heart typically occurs within the myocardium and the vessels. The amount of lipofuscin increases within myocardial fibers (not valves) with aging. Hereditary hemochromatosis is a genetic defect in iron absorption that results in extensive myocardial iron deposition (hemosiderosis). Fatty change is uncommonly seen in myocardium, but infiltration of fat cells between myofibers can occur.

4 A 23-year-old woman receiving corticosteroid therapy for an autoimmune disease has an abscess on her upper outer right arm. She undergoes minor surgery to incise and drain the abscess, but the wound heals poorly over the next month. Which of the following aspects of wound healing is most likely to be deficient in this patient? □ (A) Re-epithelialization □ (B) Fibroblast growth factor elaboration □ (C) Collagen deposition □ (D) Serine proteinase production □ (E) Neutrophil infiltration

4 (C) Glucocorticoids inhibit wound healing by impairing collagen synthesis. This is a desirable side effect if the amount of scarring is to be reduced, but it results in the delayed healing of surgical wounds. Re-epithelialization, in part driven by epidermal growth factor, is not affected by corticosteroid therapy. Angiogenesis driven by fibroblast growth factor is not significantly affected by corticosteroids. Serine proteinases are important in wound remodeling. Neutrophil infiltration is not prevented by glucocorticoids.

4 For the past week, a 61-year-old man has had increasing levels of serum AST and ALT. On physical examination, he has lower leg swelling with grade 2+ pitting edema to the knees and prominent jugular venous distention to the level of the mandible. Based on the gross appearance of the liver, seen in the figure, which of the following underlying conditions is most likely to be present? □ (A) Thrombocytopenia □ (B) Portal vein thrombosis □ (C) Chronic renal failure □ (D) Common bile duct obstruction □ (E) Congestive heart failure

4 (E) The figure shows a so-called nutmeg liver caused by chronic passive congestion. The elevated enzyme levels suggest that the process is so severe that hepatic centrilobular necrosis also has occurred. The physical findings suggest right-sided heart failure. The regular pattern of red lobular discoloration seen in the figure is unlikely to occur in hemorrhage from thrombocytopenia, characterized by petechiae and ecchymoses. A portal vein thrombus would diminish blood flow to the liver, but it would not be likely to cause necrosis because of that organ's dual blood supply. Hepatic congestion is not directly related to renal failure, and hepatorenal syndrome has no characteristic gross appearance. Biliary tract obstruction would produce bile stasis (cholestasis) with icterus.

5 A 10-year-old child developed a sore throat and fever over 24 hours. Physical examination shows pharyngeal erythema and swelling. Laboratory findings include leukocytosis. The child is given acetylsalicylic acid (aspirin). Which of the following features of the inflammatory response is most affected by this drug? □ (A) Vasodilation □ (B) Chemotaxis □ (C) Phagocytosis □ (D) Emigration of leukocytes □ (E) Release of leukocytes from bone marrow

5 (A) Aspirin (acetylsalicylic acid) blocks the cyclooxygenase pathway of arachidonic acid metabolism, which leads to reduced prostaglandin generation. Prostaglandins promote vasodilation at sites of inflammation. Chemotaxis is a function of various chemokines, and complement C3b may promote phagocytosis, but neither is affected by aspirin. Leukocyte emigration is aided by various adhesion molecules. Leukocyte release from the marrow can be driven by the cytokines Robbins & Cotran Review of Pathology Pg. 25 interleukin-1 and tumor necrosis factor.

5 A cesarean section is performed on a 20-year-old woman to deliver a term infant, and the lower abdominal incision is sutured. The sutures are removed 1 week later. Which of the following statements best describes the wound site at the time of suture removal? □ (A) Granulation tissue is still present □ (B) Collagen degradation exceeds synthesis □ (C) Wound strength is 80% of normal tissue □ (D) Type IV collagen predominates □ (E) No more wound strength will be gained

5 (A) At 1 week, wound healing is incomplete, and granulation tissue is still present. More collagen is synthesized in the following weeks. Wound strength peaks at about 80% by 3 months. Type IV collagen is found in basement membranes.

9 An experiment analyzes factors involved in the cell cycle during growth factor-induced cellular regeneration in a tissue culture. Cyclin B synthesis is induced; the cyclin B binds and activates cyclin-dependent kinase 1 (CDK1). The active kinase produced by this process is most likely to control progression in which of the following phases of the cell cycle? □ (A) G0 to G1 □ (B) G1 to S □ (C) S to G2 □ (D) G2 to M □ (E) M to G1

9 (D) CDK1 controls an extremely important transition point, the G2 to M transition during the cell cycle, which can be regulated by CDK inhibitors. The other checkpoints are regulated by a distinct set of proteins.

5 A 69-year-old woman has had transient ischemic attacks for the past 3 months. On physical examination, she has an audible bruit on auscultation of the neck. A right carotid endarterectomy is performed. The curetted atheromatous plaque has a grossly yellow-tan, firm appearance. Microscopically, which of the following materials can be found in abundance in the form of crystals that produce long, cleft-like spaces? □ (A) Glycogen □ (B) Lipofuscin □ (C) Hemosiderin □ (D) Immunoglobulin □ (E) Cholesterol

5 (E) Cholesterol is a form of lipid commonly deposited within atheromas in arterial walls, imparting a yellow color to these plaques. Glycogen is a storage form of carbohydrate seen mainly in liver and muscle. Lipofuscin is a brown pigment that increases with aging in cell cytoplasm, mainly in cardiac myocytes and in hepatocytes. Hemosiderin is a storage form of iron that appears in tissues of the mononuclear phagocyte system (e.g., marrow, liver, spleen), but can be widely deposited with hereditary hemochromatosis. Immunoglobulin occasionally may be seen as rounded globules in plasma cells (i.e., Russell bodies).

5 A 55-year-old woman has had discomfort and swelling of the left leg for the past week. On physical examination, the leg is slightly difficult to move, but on palpation, there is no pain. A venogram shows thrombosis of deep left leg veins. Which of the following mechanisms is most likely to cause this condition? □ (A) Turbulent blood flow □ (B) Nitric oxide release □ (C) Ingestion of aspirin □ (D) Hypercalcemia □ (E) Immobilization

5 (E) The most important and the most common cause of venous thrombosis is vascular stasis, which often occurs with immobilization. Turbulent blood flow may promote thrombosis, but this risk factor is more common in fast-flowing arterial circulation. Nitric oxide is a vasodilator and an inhibitor of platelet aggregation. Aspirin inhibits platelet function and limits thrombosis. Calcium is a cofactor in the coagulation pathway, but an increase in calcium has minimal effect on the coagulation process.

6 A 40-year-old man underwent laparotomy for a perforated sigmoid colon diverticulum. A wound infection complicated the postoperative course, and surgical wound dehiscence occurred. Primary closure was no longer possible, and the wound "granulated in." Six weeks later, the wound is only 10% of its original size. Which of the following processes best accounts for the observed decrease in wound size over the past 6 weeks? □ (A) Increase in synthesis of collagen □ (B) Myofibroblast contraction □ (C) Inhibition of metalloproteinases □ (D) Resolution of subcutaneous edema □ (E) Elaboration of adhesive glycoproteins

6 (B) Wound contraction is a characteristic feature of healing by second intention that occurs in larger wounds. Collagen synthesis helps fill the defect, but does not contract it. The inhibition of metalloproteinases leads to decreased degradation of collagen and impaired connective tissue remodeling in wound repair. Edema diminishes over time, but this does not result in much contraction. Adhesive glycoproteins such as fibronectin help to maintain a cellular scaffolding for growth and repair, but they do not contract.

6 A woman who is allergic to cats visits a neighbor who has several cats. During the visit, she inhales cat dander, and within minutes, she develops nasal congestion with abundant nasal secretions. Which of the following substances is most likely to produce these findings? □ (A) Bradykinin □ (B) Complement C5a □ (C) Histamine □ (D) Interleukin-1 □ (E) Phospholipase C □ (F) Platelet-activating factor □ (G) Tumor necrosis factor

6 (C) Histamine is found in abundance in mast cells, which are normally present in connective tissues next to blood vessels beneath mucosal surfaces in airways. Binding of an antigen (allergen) to IgE antibodies that have previously attached to the mast cells by the Fc receptor triggers mast cell degranulation, with release of histamine. This response causes increased vascular permeability and mucous secretions. Bradykinin, generated from the kinin system on surface contact of Hageman factor with collagen and basement membrane from vascular injury, promotes vascular permeability, smooth muscle contraction, and pain. Complement C5a is a potent chemotactic factor for neutrophils. Interleukin-1 and tumor necrosis factor, both produced by activated macrophages, mediate many systemic effects, including fever, metabolic wasting, and hypotension. Phospholipase C, which catalyzes the release of arachidonic acid, is generated from platelet activation. Platelet-activating factor (PAF) can be released by neutrophils, mast cells, monocytes, macrophages, endothelial cells, and platelets. PAF promotes vascular permeability, neutrophil aggregation, and platelet activation.

6 A 25-year-old woman who has had altered consciousness and slurred speech for the past 24 hours is brought to the emergency department. A head CT scan shows a right temporal hemorrhagic infarction. Cerebral angiography shows a distal right middle cerebral arterial occlusion. Within the past 3 years, she has had an episode of pulmonary embolism. A pregnancy 18 months ago ended in miscarriage. Laboratory studies show a false-positive serologic test for syphilis, normal prothrombin time, elevated partial thromboplastin time, and normal platelet count. Which of the following is the most likely cause of these findings? □ (A) Disseminated intravascular coagulation □ (B) Factor V mutation □ (C) Hypercholesterolemia □ (D) Lupus anticoagulant □ (E) Von Willebrand disease

6 (D) These findings are characteristic of a hypercoagulable state. The patient has antibodies that react with cardiolipin, a phospholipid antigen used for the serologic diagnosis of syphilis. These so-called antiphospholipid antibodies are directed against phospholipid-protein complexes and are sometimes called lupus anticoagulant because they are present in some patients with systemic lupus erythematosus (SLE). Lupus anticoagulant may occur in individuals with no evidence of SLE, however. Patients with lupus anticoagulant have recurrent arterial and venous thrombosis and repeated miscarriages. In vitro, these antibodies inhibit coagulation by interfering with the assembly of phospholipid complexes. In vivo, the antibodies induce a hypercoagulable state by unknown mechanisms. Disseminated intravascular coagulation is an acute consumptive coagulopathy characterized by elevated prothrombin time and partial thromboplastin time, and decreased platelet count. The prothrombin time and partial thromboplastin time are normal in patients with factor V (Leiden) mutation. Hypercholesterolemia promotes atherosclerosis over many years, and the risk of arterial thrombosis increases. Von Willebrand's disease affects platelet adhesion and leads to a bleeding tendency, not to thrombosis.

6 A 38-year-old woman experienced severe abdominal pain with hypotension and shock that led to her death within 36 hours after the onset of the pain. From the gross appearance of the mesentery, seen in the figure at the bottom of the previous column, which of the following events has most likely occurred? □ (A) Hepatitis B virus infection □ (B) Small intestinal infarction □ (C) Tuberculous lymphadenitis □ (D) Gangrenous cholecystitis □ (E) Acute pancreatitis

6 (E) The focal, chalky white deposits are areas of fat necrosis resulting from the release of pancreatic lipases in a patient with acute pancreatitis. Viral hepatitis does not cause necrosis in other organs, and hepatocyte necrosis from viral infections occurs mainly by means of apoptosis. Intestinal infarction is a form of coagulative necrosis. Tuberculosis produces caseous necrosis. Gangrenous necrosis is mainly coagulative necrosis, but occurs over an extensive area.

7 In an experiment involving observations on wound healing, researchers noted that intracytoplasmic cytoskeletal elements, including actin, interact with the extracellular matrix to promote cell attachment and migration in wound healing. Which of the following substances is most likely responsible for such interaction between the cytoskeleton and the extracellular matrix? □ (A) Epidermal growth factor □ (B) Fibronectin □ (C) Integrin □ (D) Platelet-derived growth factor □ (E) Type IV collagen □ (F) Vascular endothelial growth factor

7 (C) Integrins interact with the extracellular matrix proteins (e.g., fibronectin). Engagement of integrins by extracellular matrix proteins leads to the formation of focal adhesions at which integrins link to intracellular cytoskeletal elements such as actin. These interactions lead to intracellular signals that modulate cell growth, differentiation, and migration during wound healing. Epidermal growth factor stimulates epithelial cell and fibroblast proliferation. Platelet-derived growth factor (PDGF) can be produced by endothelium, macrophages, smooth muscle cells, and platelets; PDGF mediates migration and proliferation of fibroblasts and smooth muscle cells and migration of monocytes. Type IV collagen is found in basement membranes on which cells are anchored. Vascular endothelial growth factor promotes angiogenesis (capillary proliferation) through endothelial cell proliferation and migration in a healing response.

7 A 32-year-old woman has had a chronic cough with fever for the past month. On physical examination, she has a temperature of 37.5°C, and on auscultation of the chest, crackles are heard in all lung fields. A chest radiograph shows many small, ill-defined nodular opacities in all lung fields. A transbronchial biopsy specimen shows interstitial infiltrates with lymphocytes, plasma cells, and epithelioid macrophages. Which of the following infectious agents is the most likely cause of this appearance? □ (A) Staphylococcus aureus □ (B) Plasmodium falciparum □ (C) Candida albicans □ (D) Mycobacterium tuberculosis □ (E) Klebsiella pneumoniae □ (F) Cytomegalovirus

7 (D) These findings suggest a granulomatous inflammation, and tuberculosis is a common cause. Bacteria such as Staphylococcus and Klebsiella are more likely to produce acute inflammation. Plasmodium produces malaria, a parasitic infection without a significant degree of lung involvement. Candida is often a commensal organism in the oropharyngeal region and rarely causes pneumonia in healthy (non-immunosuppressed) individuals. Viral infections tend to produce a mononuclear interstitial inflammatory cell response.

8 A 49-year-old man is in stable condition after an infarction of the anterior left ventricular wall. He receives therapy with anti-arrhythmic and pressor agents. He develops severe breathlessness 3 days later, and an echocardiogram shows a markedly decreased ejection fraction. He dies 2 hours later. At autopsy, which of the following microscopic changes is most likely to be present in the lungs? □ (A) Congestion of alveolar capillaries with fibrin and neutrophils in alveoli □ (B) Congestion of alveolar capillaries with transudate in alveoli □ (C) Fibrosis of alveolar walls with hemosiderin-laden macrophages in alveoli □ (D) Multiple areas of subpleural hemorrhagic necrosis □ (E) Purulent exudate in the pleural space □ (F) Purulent exudate in the mainstem bronchi

8 (B) Acute left ventricular failure after a myocardial infarction causes venous congestion in the pulmonary capillary bed and increased hydrostatic pressure, which leads to pulmonary edema by transudation in the alveolar space. Neutrophils and fibrin would be found in cases of acute inflammation of the lung (i.e., pneumonia). Fibrosis and hemosiderin-filled macrophages (heart failure cells) would be found in long-standing, not acute, left ventricular failure. Subpleural hemorrhagic necrosis occurs if there are pulmonary thromboemboli. These thromboemboli can cause right-sided heart failure. Purulent exudate in the pleural space (empyema) or draining from bronchi results from bacterial infection, not heart failure.

8 A 47-year-old woman has had worsening dyspnea for the past 5 years. A chest CT scan shows panlobular emphysema. Laboratory studies show the PiZZ genotype of α1-antitrypsin (AAT) deficiency. A liver biopsy specimen examined microscopically shows abundant PAS-positive globules within periportal hepatocytes. Which of the following molecular mechanisms is most likely responsible for this finding in the hepatocytes? □ (A) Excessive hepatic synthesis of AAT □ (B) Retention of poorly folded AAT in the endoplasmic reticulum □ (C) Decreased catabolism of AAT in lysosomes □ (D) Inability to metabolize AAT □ (E) Impaired dissociation of AAT from chaperones

8 (B) Mutations in the AAT gene give rise to AAT molecules that cannot fold properly. In the PiZZ genotype, both alleles Robbins & Cotran Review of Pathology Pg. 11 have the mutation. The partially folded molecules accumulate in the endoplasmic reticulum and cannot be secreted. Impaired dissociation of the CFTR protein from chaperones causes many cases of cystic fibrosis. There is no abnormality in the synthesis, catabolism, or metabolism of AAT in patients with AAT deficiency.

8 In an experiment, release of epidermal growth factor into an area of denuded skin causes mitogenic stimulation of the skin epithelial cells. Which of the following proteins is most likely to be involved in transducing the mitogenic signal from the epidermal cell membrane to the nucleus? □ (A) G proteins □ (B) RAS proteins □ (C) Cyclin D □ (D) Cyclic AMP □ (E) Cyclin-dependent kinase

8 (B) RAS proteins transduce signals from growth factor receptors, such as epidermal growth factor, that have intrinsic tyrosine kinase activity. G proteins perform a similar function for G protein-linked, seven-spanning receptors. Cyclic AMP is an effector in the G protein signaling pathway. Cyclins and cyclin-dependent kinases regulate the cell cycle in the nucleus.

8 A 36-year-old man has had midepigastric abdominal pain for the past 3 months. An upper gastrointestinal endoscopy shows a 2-cm, sharply demarcated, shallow ulceration of the gastric antrum. A biopsy specimen of the ulcer base shows angiogenesis, fibrosis, and mononuclear cell infiltrates with lymphocytes, macrophages, and plasma cells. Which of the following terms best describes this pathologic process? □ (A) Acute inflammation □ (B) Serous inflammation □ (C) Granulomatous inflammation □ (D) Fibrinous inflammation □ (E) Chronic inflammation

8 (E) One outcome of acute inflammation with ulceration is chronic inflammation. This is particularly true when the inflammatory process continues for weeks to months. Chronic inflammation is characterized by tissue destruction, mononuclear cell infiltration, and repair. In acute inflammation, the healing process of fibrosis and angiogenesis has not begun. Serous inflammation is an inflammatory process involving a mesothelial surface (e.g., lining of the pericardial cavity), with an outpouring of fluid having little protein or cellular content. Granulomatous inflammation is a form of chronic inflammation in which epithelioid macrophages form aggregates. In fibrinous inflammation, typically involving a mesothelial surface, there is an outpouring of protein-rich fluid that results in precipitation of fibrin.

9 A 27-year-old man is on a scuba diving trip to the Caribbean and descends to a depth of 50 m in the Blue Hole off the coast of Belize. After 30 minutes, he has a malfunction in his equipment and quickly returns to the boat on the surface. He soon experiences difficulty breathing, with dyspnea and substernal chest pain, followed by a severe headache and vertigo. About 1 hour later, he develops severe, painful myalgias and arthralgias. These symptoms abate within 24 hours. Which of the following mechanisms is the most likely cause of these symptoms? □ (A) Disseminated intravascular coagulation □ (B) Systemic vasodilation □ (C) Venous thrombosis □ (D) Tissue nitrogen emboli □ (E) Fat globules in arterioles

9 (D) These findings are characteristic of decompression sickness (the "bends"). At high pressures, such as occur during a deep scuba dive, nitrogen is dissolved in blood and tissues in large amounts. Ascending too quickly does not allow for slow release of the gas, and formation of small gas bubbles causes symptoms from occlusion of small arteries and arterioles. Hemorrhage or thrombosis from disseminated intravascular coagulation is more likely to occur in underlying diseases such as sepsis, and symptoms do not abate so quickly. Systemic vasodilation is a feature of some forms of shock. Venous thrombosis is more typically a complication of stasis, which does not occur in a physically active individual. Fat globules in pulmonary arteries are a feature of fat embolism, which usually follows trauma.


Conjuntos de estudio relacionados

ch 19 - immune system - multiple choice

View Set

(18)(1)에너지관리기능사 필기 (2012년 1회 기출문제)

View Set

The tragedy of Romeo and Juliet Act II

View Set

Pregnancy, Labor, Childbirth, Postpartum - At Risk

View Set

Geometry B, Unit 10 (All lessons)

View Set

A&P Chapter 2 Chemistry Comes Alive - Practice Test

View Set